You are on page 1of 177

A

L
M

eroberogeday

lwm pln nig Esn Bisid

PaKTI 10
i

KNkmkarniBn nig eroberog


lwm pln nig Esn Bisid

KNkmkarRtYtBinitbeckeTs
elak
elak
elak
elak

lwm qun
nn; suxNa
RBwm sunit
Tit em:g

elak Gwug sMNag


elakRsI Tuy rINa
elak pl bunqay

KNkmkarRtYtBinitGkraviruT
elak lwm miKsir

karIkMuBTr
kBaa lI KuNaka

ii

elak Gwug sMNag

Garmfa
esoePAKNitviTaCMuvijBiPBelakPaKTI10 EdlGksikSakMBugkan;
enAkgdenH EckecjCabIEpkEdlEpkTI1 CakRmglMhat;eRCIserIs
EpkTI2 CaEpkdMeNaHRsay nig EpkTI3 CalMhat;Gnuvtn_ .
ral;bRbFanlMhat;nImYyenAkgesovePAenH eyIgxJM)aneRCIserIsykEt lMhat;
NaEdlmanlkNBi)ak mkeFVIdMeNaHRsayKMrUy:agek,aHk,aybMput .
eKalbMNgnkareroberogcgRkgKWedIm,ITukCaksarCMnYysRmab;Gk
sikSakgRKb;mCdan nig ma:geTotedIm,IcUlrYmelIksyvisyKNitviTakg
RbeTskm<CaeyIg[kan;EtrIkceRmInqab;rhsRsbtamsmyviTasaRsTMenIb.
esovePAenHminlhYseKhYsgenaHeT . kMhusqgedayGectnaR)akd
CamanTaMgbeckeTs nig GkraviruT . ehtuenHeyIgxJMCaGkniBn rgcaMCanicnUv
mtiriHKn;EbbsabnaBIGksikSaRKb;mCdanedaykIrIkrayedIm,IEklMGesovenH
[kan;EtmansuRkitPaBRbesIreLIgEfmeTot .
CaTIbBab; xJM)aTGkniBnsUmeKarBCUnBrGksikSaTaMgGs;mansuxPaBl
manR)aCJaQasv nig TTYl)aneCaKCyCanickgkarsikSa .
)at;dMbg fTI 27 Exmifuna qaM2011
GkniBn nig RsavRCav lwm pln
Tel : 017 768 246
Email: lim_phalkun@ymail.com
Website: www.mathtoday.wordpress.com

iii

KNitviTaCMuvijBiPBelak

eroberogeday lwm pln

- TMBr 1 -

KNitviTaCMuvijBiPBelak
CMBUkTI1

kRmglMhat;eRCIserIs
!> eK[cMnYnBit x1 , x2 , y1 , y 2 , z1 , z 2 epgpat; x1 0 , x2 0
x1y1 z12 0 nig x 2y 2 z 2 2 0 .
cUrRsayfa
8
1
1

( x1 x 2 )( y1 y 2 ) ( z1 z 2 )2 x1y1 z12 x 2y 2 z 22

( IMO 1969 )

@> eKtag I nig O erogKaCapitrgVg;carwkkgnigpitrgVg;carwkeRkA


nRtIekaN eK[RtIekaN ABC mYy .
cUrRsayfa OIA 90o luHRtaEt AB , BC , CA CasIVtnBVn .
#> eK[ n CacMnYnKt;viCmanedaydwgfa 2 2 28n2 1 CacMnYnKt;.
cUrRsayfa 2 2 28n2 1 CakaerR)akdncMnYnKt;mYy .
$> eKyk x CacMnYnBit edaydwgfa 0 x 2 .
cUrbgajfa cos2 (x) cot(x) sin 2 (x) tan( x) 1

( Baltic Way 2010 )

eroberogeday lwm pln

- TMBr 2 -

KNitviTaCMuvijBiPBelak
%> eK[RtIekaN ABC mYymanRCug a , b , c . tag r nig R erogKaCa
kaMrgVg;carwkkg nig kaMrgVg;carwkeRkAn ABC .
k> cURsayfa a cos A bosB c cos C 2Rpr
cos A cos B cos C a 2 b 2 c 2

a
b
c
2abc
r
cos A cos B cos C 1
R
abc
p
2
a 2 b 2 c 2 4 ( R r )2 A , B, C

CaknHbrimaRtnRtIekaN .
Edl
x> cUrTajbBaak;fa

CamMuRsYc.
^> eK[ f CaGnuKmn_kMNt;BIsMNMu IR eTAsMNMu IR Edlepgpat;
lkxN f (x y ) f (x).f (y ) cMeBaHRKb; x , y IR nig f '(0)
cUrkMNt;rkGnuKmn_ y f (x) .
21
&> sIVt{an } kMNt;eday a1 16
nigcMeBaH n 2 : 2a 3a 3
2
eKyk m CacMnYnKt;mYyEdl m 2 . cUrbgajfacMeBaH n m
*

eyIg)an an

3
n 3
2

1
m

n 1

n ( m 1)

m2 1
2 m
m 3
mn1

n 1

(China National Olympiad 2005)

eroberogeday lwm pln

- TMBr 3 -

KNitviTaCMuvijBiPBelak
*> eK[ a , b , c CacMnYnBitviCmanEdl a b c 3 .
a2
b2
c2
3

cUrRsaybBaak;vismPaB
2
2
2
2
ab

bc

ca

(Croatia Team Selection Tests 2011)

(>eK[ a , b , c CaRCugrbs;RtIekaNmYy . cUrRsayfa


( c a b ) 4 ( a b c )4 ( b c a ) 4

ab bc ca
a(a b c) b(b c a ) c(c a b )
(Greece National Olympiad 2007)

!0> eK[ f CaGnuKmn_kMNt;BI (0 , ) eTA IR ehIyepgpat;lkxN


f ( x ) f ( y ) f ( xy ) RKb; x 0 , y 0 nig f ' (1) .
cUrkMMNt;rkGnuKmn_ y f (x) .
!!> eKkMNt;cMnYn a0 , a1 , a2 , ...., an dUcxageRkam
1
ak 2
a 0 ; ak 1 ak
( n 1 , k 0 , 1 , 2 , ...., n 1 )
2
n
cUrbgajfa 1 n1 an 1 .
(IMO Longlists 1980)

!@> eK[ f CaGnuKmn_Cab; nig manedrIevelI IR EdlcMeBaHRKb; x, y IR


eKmanTMnak;TMng f (x y ) 1f (xf)(x)ff((yy)) nig f (x) 1 RKb; x .
cUrkMNt;GnuKmn_ y f (x) .
eroberogeday lwm pln

- TMBr 4 -

KNitviTaCMuvijBiPBelak
!#> cUrbgajfa
log a bc logb ca logc ab 4 (logab c log bc a log ca b )

cMeBaHRKb; a , b , c FMCagmYy .

( Malaysia National Olympiad 2010 )

!$> eK[rgVg;pit O kaM r nigRtIekaN ABC ERbRbYlcarwkeRkArgVg; .


bnat;kat;tam O kat;RCug AB nig AC erogKaRtg; M nig N .
kMNt;TItaMgcMNuc A nigGgt; [MN] edIm,I[pRkLaRtIekaN AMN
mantmtUcbMput ?
(RbLgsisBUEkRbcaMraCFanIPMeBj 2009)

!%> eKtag , , CargVas;mMukgRtIekaN ABC mYyEdlmanbrimaRt


2p nigkaMrgVg;carwkeRkA R .
a / cUrRsaybBaak;fa
2

R
2
2
2

cot cot cot 3 9 . 2 1

b/

etIeBlNaeTIbeK)ansmPaB ?

( Morocco National Olympiad 2011 )

eroberogeday lwm pln

- TMBr 5 -

KNitviTaCMuvijBiPBelak
3 1 1
1 0 0
!^> eK[m:aRTIs A 1 3 1 nig I 0 1 0
1 1 3
0 0 1
bgajfamansIVtcMnYnBitBIr (un ) nig ( vn ) Edlepgpat;
n IN : A n un . I v n .A

EdleKnwgbBaak;tYTUeTA

un

nig vn CaGnuKmn_n n .
!&> eK[ f CaGnuKmn_Cab; nig manedrIevelI IR EdlcMeBaHRKb; x, y IR
eKmanTMnak;TMng f ( x 2 y ) f (x)f (y ) nig f (x) 0 .
cUrkMNt;GnuKmn_ y f (x) .
!*> eK[ctuekaNe)a:g ABCD mYymanRCug AB a , BC b ,
CD c nig DA d .
P CacMnucmYyenAkgctuekaNehIy K , L , M , N CacMeNalEkg
ncMnuc P elIRCug [AB] , [BC] , [CD] , [DA] erogKa .
PL PM PN
eKdwgfa PK

h .
AB BC CD DA
k> cUrRsayfa h 2 22S 2 2 Edl S CapRkLan
a b c d
ctuekaN ABCD .
x>cUrbgajfa h 12
eroberogeday lwm pln

- TMBr 6 -

KNitviTaCMuvijBiPBelak
!(> eK[ctuekaNe)a:g ABCD mYymanRCug AB a , BC b

nig DA d . O CacMnucmYyenAkgctuekaNenHEdl
OAB OBC OCD ODA 0 90o )
k> cUrRsayfa tan a2 b24Sc2 d2
Edl S CapRkLanctuekaN ABCD .
x> kMnt; edaydwgfa tan mantmFMbMput .
CD c

@0> cMeBaHRKb; x ; y 0 cUrRsaybBaak;vismPaB


x 2 x 1 y 2 y 1 x 2 x 1 y 2 y 1 2( x y )

(Kazakhstan NMO 2010)

@!> eK[ a , b , c CacMnYnBitviCman . cUrRsaybBaak;fa


( 2a b c)2
( 2b c a)2
( 2c a b )2
2
2
8
2
2
2
2
2a (b c)
2b (c a)
2c (a b )
(USAMO 2003)

@@> cUrbgajfa

1
1 1

a b

1 1

c d
cMeBaHRKb;cMnYnBitviCman a , b , c , d

eroberogeday lwm pln

1
1

ac bd

evotNam

1962

- TMBr 7 -

KNitviTaCMuvijBiPBelak
@#> cUrbgajfa

1
1
1
4

( x y )2 ( y z )2 ( z x )2 xy yz zx

cMeBaHRKb;cMnYnBitminGviCmanxusKa x , y , z .

evotNam

2008

@$> cUrkMNt;tYTUeTAnsIVtEdlkMNt;eday
x 3, x 4 nig x x nx cMeBaHRKb; n .
7
@%> cUrkMNt;RKb;cMnYnBit x ebIeKdwgfa 128 27
.
18
6
@^> cUrKNnaplbUk
0

n 1

2
n 1

Sn

3
4
n2

...
1! 2! 3! 2! 3! 4!
n! (n 1)! (n 2)!

@&> ( China 1983 )


eK[ f CaGnuKmn_kMNt;elIcenaH [0 ,1] edaydwgfa
f (0) f (1) 1 nig | f (a) f (b) | | a b |
cMeBaHRKb; a b kgcenaH [0 ,1] .
cUrbgajfa | f (a) f (b) | 12 .
@*> ( AIME 1988 )
cUrkMNt;KUncMnYnKt; (a ,b) edaydwgfa ax bx 1
Eckdac;nwg x x 1 .
17

16

eroberogeday lwm pln

- TMBr 8 -

KNitviTaCMuvijBiPBelak
@(> edaHRsaykgsMNMucMnYnBitnsmIkar
x 3x x 2 .
#0> ( Korean Mathematics Competition 2000 )
cUrkMNt;RKb;cMnYnBit x Edlepgpat;smIkar
3

2x 3x 4x 6x 9x 1

#!> ( China 1992 )


cUrbgajfa 16

1
17
k 1
k
z1 z 2 z 2 2
#@> eK[ z1 , z 2 , z 3 CacMnYnkMupicEdl z12 z 22 z 32 3
z z z 4
1 2 3
cUrKNnatm S z z 1z 1 z z 1z 1 z z 1z 1
1 2
3
2 3
1
3 1
2

80

##> eK[ x , y , z CabIcMnYnBitviCmanEdl x y z 1 .


cUrkMNt;tmtUcbMputn 1 x x 1 y y 1 z z .
#$> ( Iran 1996 )
eK[bIcMnYnBitminGviCman a , b , c nigminsUnRBmKaBIr .
cUrRsaybBaak;fa
4

1
1
1 9
(ab bc ca)

2
2
2
(a b) (b c) (c a) 4

eroberogeday lwm pln

- TMBr 9 -

KNitviTaCMuvijBiPBelak
#%> eK[ a , b , c CabIcMnYnBitminGviCman .
cUrRsayfa a3 b3 c3 3abc 2 ( b 2 c a)3 .
#^> eK[RtIekaN ABC mYymanRCug BC a , CA b , AB c
ehIymMukg A , B , C CamMuRsYcbmMuEkg .
tag S CapRkLan ABC
cUrRsaybBaak;fa 14 14 14 9 2 .
a

16 S

#&> Turkey 2007


eK[bIcMnYnBitviCman a , b , c Edl a b c 1 .
cUrRsaybBaak;fa
1
1
1
1

ab 2c2 2c bc 2a2 2a ca 2b2 2b ab bc ca


#*> eK[ a , b , c CabIcMnYnBitviCman .

cUrRsaybBaak;fa
a2
2a2 ab b2

b2
2b2 bc c2

#(> eK[ a , b , c CabIcMnYnBitxusKa .


a2
b2
cUrRsayfa

2
2
(b c )

eroberogeday lwm pln

(c a )

c2
2c2 ca a2

c2
2
2
(a b )

abc
2

- TMBr 10 -

KNitviTaCMuvijBiPBelak
$0> cUrRsaybBaak;vismPaB
a 2 (1 b )2 b 2 (1 c)2 c2 (1 a )2

3 2
2

cMeBaHRKb;cMnYnBit a , b , c .
$!> eK[RtIekaN ABC mYyEkgRtg; C . D nig E CacMNucBIreRCIserIs
enAelIGIub:UetnUs Edl BC BD nig AC AE .
F nig G CacMeNalEkgn D nig E elIRCug AC nig BC erogKa .
cUrRsaybBaak;fa DE DF EG .
$@> eKyk D CacMnucmYynRCug BC rbs;RtIekaN ABC ehIy E
nig F CacMeNalEkgn B nig C elI AD .
ebI R CacMnuckNaln BC enaHRsayfa RE RF
$#> eK[kaer ABCD mYy . E CaRbsBVrvagGgt;RTUgTaMgBIr
ehIy N CacMnucmYysitelI AE .
cUrRsayfa AB2 BN 2 AN.NC nig AN 2 NC2 2BN 2
$$> eK[ ABC CaRtIekaNmYyehIy D CaeCIgnkm<s;KUsBIkMBUl A .
yk E nig F sitenAelIbnat;kat;tam D edaydwgfa AE Ekgnwg BE
ehIy AF Ekgnwg CF Edl E nig F xusBI D .
yk M nig N CacMNuckNalnGgt; BC nig EF erogKa .
cUrRsayfa AN Ekgnwg NM ?
eroberogeday lwm pln

- TMBr 11 -

KNitviTaCMuvijBiPBelak
$%> rkGnuKmn_ f : IR IR Edlepgpat;
x, y IR : f ((x y)2 ) x 2 2yf (x) f (y)

$^> cMeBaHRKb;cMnYnBitviCman a , b ,c cUrRsaybBaak;fa


2
2
2
2
2
2
(

)(

)(

) 3
a
b
b
c
c
a
3 (a b)(b c)(c a ) 6
abc
8

$&> Greece National Olympiad 2011


eK[ a , b , c CacMnYnBitviCmanEdlmanplbUkesI 6 .
cUrkMNt;tmGtibrman
S 3 a 2 2bc 3 b 2 2ca 3 c 2 2ab

$*>USAMO 1989
cMeBaHRKb;cMnYnKt;viCman n eK[
1 1
1
....
2 3
n
Tn S1 S 2 S 3 .... Sn

Sn 1

Un

T1 T2 T3
T

.... n
2
3
4
n1

cUrkMNt;edayeFIVdMeNaHRsay nUvcMnYnKt; 0 a, b, c, d 1000 000


edaydwgfa T1988 aS1989 b nig U1988 cS1989 d .

eroberogeday lwm pln

- TMBr 12 -

KNitviTaCMuvijBiPBelak
$(>(Japan Mathematical Olympiad Finals 2010)
eK[ x , y , z CacMnYnBitviCman . cUrbgajfa
1 yz zx 1 zx xy 1 xy yz

1
(1 x y)2 (1 y z)2 (1 z x)2

%0> China Team Selection Test 2002


eK[sIVt a 1 , a 5 nig a
1

n 1

ana n 1
an a
2

cUrkMNt;tYTUeTAnsIVt {a } CaGnuKmn_n n .
%!> China Team Selection Test 2006
eK[ x , x ,...., x CacMnYnBitviCmanEdl x
cUrRsayfa

1
n

.
x

2
n 1

n 2

i 1

i 1

1 .

i 1

1 xi

n1

%@> China Team Selection Test 2005


eK[ a , b ,c CabIcMnYnBitminGviCman Edl ab bc ca 13 .
cUrbgajfa a

1
1
1

3
2
2
2
bc 1 b ca 1 c ab 1

eroberogeday lwm pln

- TMBr 13 -

KNitviTaCMuvijBiPBelak

eroberogeday lwm pln

- TMBr 14 -

KNitviTaCMuvijBiPBelak
CMBUkTI2

EpkdMeNaHRsay
lMhat;TI1
eK[cMnYnBit x1 , x2 , y1 , y 2 , z1 , z 2 epgpat; x1 0 , x2 0
x1y1 z12 0 nig x 2y 2 z 2 2 0 .
cUrRsayfa
8
1
1

( x1 x 2 )( y1 y 2 ) ( z1 z 2 )2 x1y1 z12 x 2y 2 z 2 2

( IMO 1969 )

dMeNaHRsay
Rsayfa
8
(x1 x2 )(y1 y2 ) (z1 z2 )

x1y1 z1

1
x2y2 z2

(*)

tag a x1y1 z12 0 ; b x2y 2 z 22 0


nig c (x1 x2 )(y1 y 2 ) (z1 z 2 )2
a z 12
eday x1 0 nig x2 0 enaH y1 x 0 nig y 2 0
1

eroberogeday lwm pln

- TMBr 15 -

KNitviTaCMuvijBiPBelak
eKman
c ( x1y1 z12 ) ( x 2y 2 z 2 2 ) x1y 2 x 2y1 2z1z 2
c a b x1y 2 x 2y1 2z1z 2

eday a x1y1 z1 enaH

a z 12
x1
y1

nig

b z 22
x2
y2

b x2y 2 z 2

enaH

a z 12
b z 22
eK)an c a b y 2 ( y ) y1 ( y ) 2z1z 2
1
2
y
y
y
y
a b 2 a 1 b 2 z 12 2 z 1z 2 1 z 2 2
y1
y2
y1
y2
y

y
y
y1
a b 2 a 1 b 2 z1
z 2
y1
y2
y2
y1

eday yy2 z1 yy1 z 2 0 enaHeKTaj)an


2
1

y
y
y
y
c a b 2 a 1 b eday 2 a 1 b 2 ab
y1
y2
y1
y2
enaH c a b 2 ab ( a b )2 (1)
eyIg]bmafavismPaB (*) BiteBalKW 8c a1 b1 Bit

eK)an c a8abb eday c (


eroberogeday lwm pln

a b )2

tamvismPaB (1)
- TMBr 16 -

KNitviTaCMuvijBiPBelak
eyIgnwgRsayfa (
smmUl (a b)(

8ab
ab
a b )2 8ab
a b )2

tamvismPaB AM GM eKman
a b 2 ab ehIy ( a b )2 4
enaH (a b)( a b ) 8ab Bit
dUcenH

ab

8
1
1

( x1 x2 )(y1 y 2 ) ( z1 z 2 )2 x1y1 z12 x2y 2 z 22

vismPaBenHkaysmPaBluHRtaEt
y2
y
y
y
z1
z 2 1 b z1y 2 z 2y1 nig 2 a 1 b .
y
y
y
y
1

eroberogeday lwm pln

- TMBr 17 -

KNitviTaCMuvijBiPBelak
lMhat;TI2
eKtag I nig O erogKaCapitrgVg;carwkkgnigpitrgVg;carwkeRkA
nRtIekaN eK[RtIekaN ABC mYy .
cUrRsayfa OIA 90o luHRtaEt AB , BC , CA CasIVtnBVn .
dMeNaHRsay
Rsayfa OIA 90o luHRtaEt AB , BC , CA CasIVtnBVn
C

I
O
A

tag BC a , AC b , AB c nig p a b2 c
ehIy r nig R CakaMrgVg;carwkkg nig carwkeRkAnRtIekaN ABC .
yk H CacMeNaln I elI [AC] enaHeK)an
IH r nig AH p a .
eroberogeday lwm pln

- TMBr 18 -

KNitviTaCMuvijBiPBelak
-sntfa OIA 90o enaH OA 2 OI 2 IA 2
tamRTwsIbTGWEleKman OI 2 R(R 2r )
tamRTwsIbTBItaKrkgRtIekaNEkg AHI : IA 2 AH 2 IH 2
eK)an R 2 R(R 2r ) r 2 (p a)2
b 2rR r 2 (p a)2
tamrUbmnehr:ug S pr p(p a)(p b)(p c) abc
4R
eKTaj
eK)an

abc
(p a )(p b )(p c)
ni
g
r2
2p
p
abc (p a )(p b )(p c)

(p a )2
2p
p
2rR

abc 2(p a )(p b )(p c) 2p(p a )2


abc 2(p a )[(p b )(p c) p(p a )]
abc ( 2p 2a )(p 2 pb pc bc p 2 pa )
abc ( 2p 2a )[2p 2 p(a b c) bc]
abc (b c a )( 2p 2 2p 2 bc )
abc bc(b c a )
abca
eKTaj 2a b c enaH c , a , b

CasIVtnBVn .
-sntfa c , a , b CasIVtnBVnenaHeK)an 2a b c
tamRTwsIbTkUsIunUskgRtIekaN OIA eK)an
eroberogeday lwm pln

- TMBr 19 -

KNitviTaCMuvijBiPBelak
OA 2 OI 2 IA 2 2OI .IA cos OIA
OI 2 IA 2 OA 2
eKTaj cos OIA 2OI.IA
R( R 2r ) r 2 (p a )2 R 2

2OI .IA

r 2 2rR (p a )2

2OI .IA
a b c 3a
eday b c 2a enaH p 2 2
3a
3a
3a
( a )( b )( c) a( 3a 2b )( 3a 2c)
2
2
r2 2

3a
12a
2
9a 2 6(b c)a 4bc 4bc 3a 2

12
12
bc

ehIy R 2rR abc


3a
2( ) 3
2
4bc 3a 2 bc 3a
( a )2
0
eK)an cos OIA 12 2OI3.IA 2
eK)an OIA 90o .

dUcenH

OIA 90o

eroberogeday lwm pln

luHRtaEt

AB , BC , CA

CasIVtnBVn .
- TMBr 20 -

KNitviTaCMuvijBiPBelak
lMhat;TI3
eK[ n CacMnYnKt;viCmanedaydwgfa 2 2 28n2 1 CacMnYnKt;.
cUrRsayfa 2 2 28n2 1 CakaerR)akdncMnYnKt;mYy .
dMeNaHRsay
rebobTI1
Rsayfa 2 2 28n2 1 CakaerR)akdncMnYnKt;
tambRmab; 2 2 28n2 1 CacMnYnKt;naM[man m IN Edl
28n 2 1 m 2 b m 2 28n 2 1 CasmIkar Pell .
KUcemIydMbUgnsmIkarenHKW m 127 , n 24
eRBaH 1272 28 242 1 . cMeBaHRKb; k 1eKGacsresr
m 2 28n 2 127 2 28 242 (127 28 242 )k
(m 2 7n )(m 2 7n ) (127 48 7 )k (127 48 7 )k
m 2 7n (127 48 7 )k
eKTaj
m 2 7n (127 48 7 )k

eK)ancemIyTUeTAnsmIkar
(127 48 7 )k (127 48 7 )k
m
2
(127 48 7 )k (127 48 7 )k
n
4 7

eroberogeday lwm pln

- TMBr 21 -

KNitviTaCMuvijBiPBelak
kgkrNIenHeK)an

2 2 28n 2 1 2 2m

2 2 28n 2 1 2 (127 48 7 )k (127 48 7 )k

eday 127 48 7 (8 3 7 )2
nig (8 3 7 )(8 3 7 ) 1 enaHeK)an

k 2

2 (127 48 7 ) (127 48 7 ) (8 3 7 ) (8 3 7 )
k

eK)an 2 2 28n2 1 [(8 3 7 )k (8 3 7 )k ]2


CakaerR)akdncMnYnKt;eRBaH (8 3 7 )k (8 3 7 )k CacMnYnKt;
dUcenHebI 2 2 28n2 1 CacMnYnKt;enaH 2 2 28n2 1
CakaerR)akdncMnYnKt; .
rebobTI2
Rsayfa 2 2 28n2 1 CakaerR)akdncMnYnKt;
tambRmab; 2 2 28n2 1 CacMnYnKt;naM[man m IN Edl
28n 2 1 m 2 b m 2 1 28n 2
b m2 1 . m2 1 7n2
tamsmIkarenHeKTaj)an

m 1
2
p

m 1 7q 2
2

Edl p nig q CacMnYnKt;viCman .


eroberogeday lwm pln

m 1
2
7
p

m 1 q2
2

- TMBr 22 -

KNitviTaCMuvijBiPBelak
eKTaj)an
m 2p 2 1 , m 14q 2 1 b m 14p 2 1 , m 2q 2 1

-krNI m 2p2 1 , m 14q2 1


eK)an 2p2 1 14q2 1 b p2 7q2 1 CasmIkarKan
cemIykg IN eRBaHGgTIBIrsmIkarEcknwg 7 [sMNl; 1
EtGgTIBIrnsmIkarEcknwg 7 minGac[sMNl; 1 eT eRBaHRKb;
cMnYnKt;viCman p cMnYn p2 Ecknwg 7 [sMNl; 1 , 2 , 4 .
-krNI m 14p2 1 , m 2q2 1
eK)an 14p2 1 2q2 1 b q2 7p2 1 CasmIkarmancemIy
kgsMNMu IN eRBaH q2 7p2 Ecknwg 7 Gac[sMNl; 1 .
ehtuenH manKU p , q IN Edl m 14p2 1 , m 2q2 1
kgkrNIenHeK)an 2 2 28n2 1 2 2m
cMeBaH m 2q2 1 eK)an
2 2 28n 2 1 2 2( 2q 2 1) 4q 2 CakaerR)akd .
beKGacyk m 14p2 1 eK)an
2 2 28n 2 1 2 2(14p 2 1) 28p 2 4

CakaerR)akd
b 7p2 1 q2 .

4(7p 2 1) 4q 2

eRBaH q2 7p2 1
eroberogeday lwm pln

- TMBr 23 -

KNitviTaCMuvijBiPBelak
rebobTI3 Rsayfa 2 2 28n2 1 CakaerR)akdncMnYnKt;
tambRmab; 2 2 28n2 1 CacMnYnKt;naM[mancMnYnKt;viCman p
Edl 28n2 1 (2p 1)2 4p2 4p 1
eK)an p(p 1) 7n2 eday GCD(p , p 1) 1
eRBaH (p 1) p 1 tamRTwsIbT Bezout
eKTaj)an p Eckdac;nwg 7 b p 1 Eckdac;nwg 7 .
-krNI p Eckdac;nwg 7
tam p(p 1) 7n2 eKTaj)an p 7k 2 , p 1 t 2
RKb;cMnYnKt;viCman k nig t ehIy GCD(k , t ) 1 .
eKman 7k 2 1 t 2 b t 2 7k 2 1 CasmIkarmanbskg IN
eRBaH t 2 7k 2 Ecknwg 7 mansMNl; 1 , 2 b 4 .
-krNI p 1 Eckdac;nwg 7
tam p(p 1) 7n2 eKTaj)an p k 2 , p 1 7t 2
RKb;cMnYnKt;viCman k nig t ehIy GCD(k , t ) 1 .
eKman k 2 1 7t 2 b k 2 7t 2 1 CasmIkarKanbskg IN
eRBaH k 2 7t 2 Ecknwg 7 minGacmansMNl; 1 eT .
cMeBaH p 7k 2 , p 1 t 2 Edl t 2 7k 2 1 eKman
2 2 28n 2 1 2 2( 2p 1) 4p 4 4t 2 Cakaer .
eroberogeday lwm pln

- TMBr 24 -

KNitviTaCMuvijBiPBelak
lMhat;TI4
eKyk x CacMnYnBit edaydwgfa 0 x 2 .
cUrbgajfa cos2 (x) cot(x) sin 2 (x) tan( x) 1
( Baltic Way 2010 )

dMeNaHRsay
bgajfa cos2 (x) cot(x) sin 2 (x) tan( x) 1
x
sin x
tan x
edayCMnYs cot x cos
ni
g
vi
s
mPaBsmmU
l
sin x
cos x
cos 3 x sin 3 x

1 ehIy sin x 0 , cos x 0 x (0, )


sin x
cos x
2
tamvismPaB AM GM eK)an
cos 3 x cos 3 x

sin 2 x 3 cos2 x
sin x
sin x
cos 3 x 3 cos 2 x sin 2 x
b sin x
(1)
2
sin 3 x sin 3 x
ehIy cos x cos x cos2 x 3 sin 2 x
sin 3 x 3 sin 2 x cos 2 x
b cos x
( 2)
2

eroberogeday lwm pln

- TMBr 25 -

KNitviTaCMuvijBiPBelak
bUkvismPaB (1) nig (2) GgnigGgeK)an
cos 3 x sin 3 x

cos 2 x sin 2 x 1 Bit


sin x
cos x
dUcenH cos2 (x) cot(x) sin 2 (x) tan( x) 1 .

eroberogeday lwm pln

- TMBr 26 -

KNitviTaCMuvijBiPBelak
lMhat;TI5
eK[RtIekaN ABC mYymanRCug a , b , c . tag r nig R erogKaCa
kaMrgVg;carwkkg nig kaMrgVg;carwkeRkAn ABC .
k> cURsayfa a cos A bosB c cos C 2Rpr
cos A cos B cos C a 2 b 2 c 2

a
b
c
2abc
r
cos A cos B cos C 1
R
abc
p
2
a 2 b 2 c 2 4 ( R r )2 A , B, C

Edl
CaknHbrimaRtnRtIekaN .
x> cUrTajbBaak;fa

dMeNaHRsay
k> Rsayfa a cos A bosB c cos C 2Rpr

CamMuRsYc.

M
O

eroberogeday lwm pln

- TMBr 27 -

KNitviTaCMuvijBiPBelak
eKman BAC 12 BOC BOK mMupit nig mMucarwkkgrgVg;
OK
kgRtIekaNEkg OKB eKman cos BOK OK

R
OB
eKTaj OK R cos BOK R cos A .
RsaydUcKaEdr OL R cos B , OM R cos C
tag S CapRkLanRtIekaN ABC enaHeK)an
S SOBC SOCA SOAB
1
1
1
pr BC.OK CA.OL AB.OM
2
2
2
1
1
1
pr a R cos A bR cos B cR cos C
2
2
2
1
pr R(a cos A b cos B c cos C)
2
2pr
a cos A b cos B c cos C
R
a 2 b 2 c 2 2bc cos A

dUcenH
tamRTwsIbTkUsIunUseKman
c a
eKTaj cosa A b 2abc
.
RsaydUcKaEdreK)an
cos B a c b
cos C a

ni
g
b
2abc
c
2

b 2 c2
2abc
cos A cos B cos C a 2 b 2 c 2

a
b
c
2abc
2

dUcenH

eroberogeday lwm pln

.
- TMBr 28 -

KNitviTaCMuvijBiPBelak
ma:geToteKman
b 2 c 2 a2 c 2 a 2 b 2 a 2 b 2 c 2

cos A cosB cosC


2bc
2ac
2ab
a(b2 c2 ) b(c2 a2 ) c(a2 b2 ) (a3 b3 c3 )

2abc
(a b c)(a2 b2 c2 ) 2(a3 b3 c3 )

(*)
2abc

tamrUbmnehr:ugeKman p(p a)(p b)(p c) pr


elIkGgTaMgBIrCakaereK)an
p(p a )(p b )(p c) p 2r 2
(p a )(p b )(p c) pr 2

p 3 (a b c)p 2 (ab bc ca)p abc pr 2


abc
a b c 2p
abc 4Rpr
S
pr
4R
p 3 2p 3 (ab bc ca)p 4Rpr pr 2

eday
ehIy
enaH
eK)an
eKTaj ab bc ca p r 4rR
eday (a b c) a b c 2(ab bc ca)
eK)an a b c 4p 2(p r 4rR )
b a b c 2(p r 4rR )
ehIy a b c 3abc (a b c)(a b c ab bc ca)
eKTaj)an a b c 2(p 3pr 6Rpr )
2

eroberogeday lwm pln

- TMBr 29 -

KNitviTaCMuvijBiPBelak
TMnak;TMng (*) Gacsresr
4p(p 2 r 2 4Rr ) (p 3 3pr 2 6Rpr )
cos A cos B cos C
8Rpr
p 2 r 2 4Rr p 2 3r 2 6Rr

2Rr
2r 2 2Rr r

1
2Rr
R

dUcenH cos A cos B cos C 1 Rr .


x> TajbBaak;fa a b c 4 (R r )
tamvismPaB Cauchy Schwarz eKman
2

( x1y1 x 2 y 2 x 3 y 3 )2 ( x1 x 2 x 3 )( y 1 y 2 y 3 )
2

yk x
nig y

a cos A , x 2 b cos B , x 3 c cos C

cos C
cos B
cos A

,
z
,
y
1
2
2
c
b
a
2
2
2pr a b c 2
2
(cos A cos B cos C)
.
R
2abc
r 2 2pr a 2 b 2 c 2
(1 )
.
R
R
8Rpr

dUcenH

eK)an

(r R ) 2 a 2 b 2 c 2

2
R
4R 2
a 2 b 2 c 2 4 ( R r )2

eroberogeday lwm pln

- TMBr 30 -

KNitviTaCMuvijBiPBelak
lMhat;TI6
eK[ f CaGnuKmn_kMNt;BIsMNMu IR eTAsMNMu IR* Edlepgpat;
lkxN f (x y ) f (x).f (y ) cMeBaHRKb; x , y IR nig f '(0)
cUrkMNt;rkGnuKmn_ y f (x) .
dMeNaHRsay
kMNt;rkGnuKmn_ y f (x)
eKman f (x y ) f (x).f (y ) cMeBaHRKb; x , y IR
yk x y 0 eK)an f (0) f (0)
naM[ f (0)[1 f (0)] 0 eKTaj f (0) 0 b f (0) 1
eday f CaGnuKmn_kMNt;BIsMNMu IR eTAsMNMu IR* enaH f (0) 0
eK)an f (0) 1 .
eFIVedrIeveFobnwg x kgsmPaB f (x y ) f (x).f (y ) edaycat;Tuk y
CaGefrminGaRsynwg x eK)an f '(x y ) f '(x)f (y )
yk x 0 eK)an f '(y ) f '(0)f (y ) eday f '(0)
eKTaj)an f '(y ) f (y ) 0 CasmIkarDIepr:g;EsllIenEGlMdab;TI1
eK)ancemIyTUeTAnsmIkarrag f (y ) k e
cMeBaH y 0 eK)an f (0) k 1 eRBaH f (0) 1
dUcenH f (x) e CaGnuKmn_RtUvrk .

eroberogeday lwm pln

- TMBr 31 -

KNitviTaCMuvijBiPBelak
lMhat;TI7
21
sIVt {an } kMNt;eday a1 16
nigcMeBaH n 2 : 2an 3an1 n31
2
eKyk m CacMnYnKt;mYyEdl m 2 . cUrbgajfacMeBaH n m
eyIg)an an

2n 3

n ( m 1)

m2 1
2 m
m 3
mn1

1
m

(China National Olympiad 2005)

dMeNaHRsay
bgajfa

an n 3
2

eKman 2an 3an 1

1
m

n ( m 1)

m2 1
2 m
mn1
m 3

3
2n 1

naM[ 2n an 3.2n 1 an 1 43
n

n 1

b 23 an 23 an 1 43 1n
3


k 1
n 2 k
3 n 1

2
eK)an 3 ak 3 ak 1 4 k

k 2 3
k 2

eroberogeday lwm pln

- TMBr 32 -

KNitviTaCMuvijBiPBelak
n 1

1
1


n
2
3 1
2
3
an a1 2 .
1
3
4 3
3
1
3
n
2 21 1
1
2
an . 1 n 1
3 16 8
3
3
n
n
3
3
3
3
eKTaj)an an 2 n 3 b an n 3 2


2
2
1
n ( m 1)
tag P an n3 3 m m 23 m
2

n
n ( m 1)
3 m
2 m
eK)an P 2 m 3

m2 1
m 1
1)
eKman m n 1 (m(m 1)(1m

n
)n
1
m1
m2 1
edIm,IRsay[eXIjfa P m n 1
n
eyIgRtUvRsay[eXIjfa (1 m 1) P m 1
tamvismPaB Bernoully eKman

eroberogeday lwm pln

- TMBr 33 -

KNitviTaCMuvijBiPBelak

n
n
1 n m
1

(1
1
)

m1
m1
m 1
1 1

m
mn

n
1
1

1
naM[ m 1

1 m

1 1
(1 )

m
m
cMeBaHRKb; m 2 tameTVFajtun eKman
1

enaH 1 m

1 2
1
1 1
Cm 2 Cm
.... m Cm
m
m
m
m

1 2 5 1
9 3
1
1 C1m 2 Cm


2 2m 4 2
m
m

eKTaj)an 1 mn 1

2

3

2n
2m

eKTaj 1 mn 1 P 3

2n

2n
2 m

n

m 1 3

n ( m 1)

Et P 2 m 23 m

n
n
( m 1)

n
2 m
2 m
eK)an (1 m 1) P 3 m 3

n
3 m

eroberogeday lwm pln

- TMBr 34 -

KNitviTaCMuvijBiPBelak
n
2 m

]bmafa 3

( m 1)

2 m

m
m1

Bit

n
2 m

edaytag u 3 enaH u(m um 1 ) m 1



smmUl mu um m 1 0
m(u 1) (um 1) 0
(u 1)[m (um 1 .... u 1)] 0
n
2 m

eday m n & m 2 enaH 0 u 3 1



naM[ (u 1)[m (um 1 .... u 1)] 0 Bit
dUcenH a 3 m 2 m 1
n ( m 1)
m

1
m

2n 3

eroberogeday lwm pln

mn1

- TMBr 35 -

KNitviTaCMuvijBiPBelak
lMhat;TI8
eK[ a , b , c CacMnYnBitviCmanEdl a b c 3 .
a2
b2
c2
3
cUrRsaybBaak;vismPaB

2
2
2
2
ab

bc

ca

(Croatia Team Selection Tests 2011)

dMeNaHRsay
a2
b2
c2
3
RsaybBaak;vismPaB

a b 2 b c2 c a2 2
a2
a(a b 2 ) ab 2
ab 2
a
(1)
eKman

2
2
2
ab
ab
ab
b2
bc 2
c2
ca2
dUcKaEdr 2 b 2 (2) ; 2 c 2 (3)
bc
bc
ca
ca
a2
b2
c2
tag S 2 2 2 . bUkvismPaB (1), (2) & (3)
ab
bc
ca
ab 2
bc 2
ca2
eK)an S 3 ( 2 2 2 )
ab
bc
ca
edIm,IRsayfa S 32 enaHeyIgnwgRsayfa
ab 2
bc 2
ca2
3 3
3
S
3

.
2
2
2
2
2
ab
bc
ca
2ab 2
2
tamvismPaB AM GM eKman a b 2b a b a

eroberogeday lwm pln

- TMBr 36 -

KNitviTaCMuvijBiPBelak
ab 2
b a
bc 2
c b ca2
a c
,
eKTaj

ehI
y

2
2 c a2
2
a b2
b c2
ab 2
bc 2
ca2
1
ehtuenH

(a c b a c b ) (*)
2
2
2
2
ab
bc
ca
tamvismPaB Cauchy Schwarz eKman
(a c b a c b )2 (a b c)(ab bc ca)
(a c b a c b )2 3(ab bc ca )

ehIy (ab bc ca)2 (a2 b2 c2 )(b2 c2 a2 )


(ab bc ca)2 (a 2 b 2 c 2 )2
ab bc ca a 2 b 2 c2
ab bc ca (a b c)2 2(ab bc ca)
3(ab bc ca) (a b c)2 9

eKTaj (a c b a c b )2 3(ab bc ca) 9


naM[ a c b a c b 3 (**)
tamvismPaB (*) & (**) eKTaj)an
ab 2
bc 2
ca2
3

Bi
t
2
2
2
2
ab

dUcenH

bc
ca
a2
b2
c2
3

a b 2 b c2 c a2 2

eroberogeday lwm pln

- TMBr 37 -

KNitviTaCMuvijBiPBelak
lMhat;TI9
eK[ a , b , c CaRCugrbs;RtIekaNmYy . cUrRsayfa
( c a b ) 4 ( a b c )4 ( b c a ) 4
ab bc ca

a(a b c) b(b c a ) c(c a b )

dMeNaHRsay
Rsayfa

(Greece National Olympiad 2007)

( c a b ) 4 ( a b c )4 ( b c a ) 4

ab bc ca
a(a b c) b(b c a ) c(c a b )

tamvismPaB AM GM eKman
( c a b )4
a(a b c) 2(c a b )2
a( a b c )

eKTaj

( c a b )4
2(c a b )2 a(a b c)
a( a b c )

( c a b )4
a 2 2b 2 2c 2 5ac 5ab 4bc (1)
a( a b c )

b
RsaydUcKaEdreK)an

( a b c )4
2a 2 b 2 2c 2 5ab 5bc 4ac ( 2)
b(b c a )
( b c a )4
2a 2 2b 2 c 2 5bc 5ac 4ab ( 3)
c(c a b )

eroberogeday lwm pln

- TMBr 38 -

KNitviTaCMuvijBiPBelak
bUkvismPaB (1) ,

( 2) & ( 3)

eK)an

S 5(a 2 b 2 c 2 ) 4(ab bc ca) ab bc ac

dUcenH

Bit

(c a b)4 (a b c)4 (b c a)4


S

ab bc ca
a(a b c) b(b c a) c(c a b)

eroberogeday lwm pln

- TMBr 39 -

KNitviTaCMuvijBiPBelak
lMhat;TI10
eK[ f CaGnuKmn_kMNt;BI (0 , ) eTA IR ehIyepgpat;lkxN
f ( x ) f ( y ) f ( xy ) RKb; x 0 , y 0 nig f ' (1) .
cUrkMMNt;rkGnuKmn_ y f (x) .
dMeNaHRsay
kMMNt;rkGnuKmn_ y f (x)
eKman f (x) f (y ) f (xy) RKb; x 0 , y 0
yk y 0 eK)an f (x) f (1) f (x) naM[ f (1) 0
eFIVedrIeveFobnwg x kgsmPaB f (x) f (y ) f (xy) edaycat;Tuk y
CaGefrminGaRsynwg x eK)an f '(x) y f '(xy)
yk x 1 eK)an f '(1) yf '(y ) eday f '(1) enaH f '(y ) y
eKTaj)an f (y ) y dy ln | y | k
yk y 1 enaH f (1) k 0 eRBaH f (1) 0 .
ehtuenH f (y ) ln | y | ln y RKb; y 0
dUcenH f (x) . ln x CaGnuKmn_RtUvrk .

eroberogeday lwm pln

- TMBr 40 -

KNitviTaCMuvijBiPBelak
lMhat;TI11
eKkMNt;cMnYn a0 , a1 , a2 , ...., an dUcxageRkam
1
ak 2
a 0 ; ak 1 ak
( n 1 , k 0 , 1 , 2 , ...., n 1 )
n
2
cUrbgajfa 1 n1 an 1 .
(IMO Longlists 1980)

dMeNaHRsay
bgajfa 1 n1 an 1
ak 2 a k (n a k )
eKman ak 1 ak n n
eK)an a 1 a (an n) a1 a 1 n
k 1
k k
k
k
b a 1 n a1 a 1
k
k
k 1
n 1
n 1 1
1
ehtuenH a n ( a a 1 ) a1 a1 2 a1 (*)
k 0 k
k 0 k
k 1
0
n
n
eKman a0 12 0 Bit . ]bmafa ak 0 Bit
ak 2
tam ak 1 ak n eKTaj)an ak 1 0 Bit

eroberogeday lwm pln

- TMBr 41 -

KNitviTaCMuvijBiPBelak
1
1
, n 1
k n n
1 1 1
1 n
... 1
n
n
n
n n

dUcenH ak 0 enaH ak n n b a
eK)an

n 1

k 0

n 1
1

ak n k 0

(n )

tam (*) eKTaj)an 2 a1

naM[ an 1

(i )

ma:geToteday an 1 enaH ak 1 b ak n n 1
b a 1 n n 1 1 RKb; k 0 , 1 , 2 , ...., n 1 .
k

n 1

n 1

1
n

k 0
k 0 n 1 n 1
edayBiniteXIjfaRKb; n 1 eKman n n 1 nn 12 22 0
n 1
n 1
enaHeKTaj)an a 1 n n n 1 nn 12
k 0 k
tam (*) eKTaj)an 2 a1 nn 12
n
b a1 2 nn 12 n n 1 enaH an n n 1 1 n1 (ii )
n
tam (i ) & (ii ) eKTaj)an 1 n1 an 1 .
dUcenH 1 n1 an 1 .

eK)an

ak n

eroberogeday lwm pln

- TMBr 42 -

KNitviTaCMuvijBiPBelak
lMhat;TI12
eK[ f CaGnuKmn_Cab; nig manedrIevelI IR EdlcMeBaHRKb; x, y IR
eKmanTMnak;TMng f (x y ) 1f (xf)(x)ff((yy)) nig f (x) 1 RKb; x .
cUrkMNt;GnuKmn_ y f (x) .
dMeNaHRsay
kMNt;GnuKmn_ y f (x)
eKman f (x y ) 1f (xf)(x)ff((yy))
yk y 0 eK)an f (x) 1f (xf)(x)ff((00))
b f (x) f (x)f (0) f (x) f (0)
b f (0)[f (x) 1] 0 eKTaj)an f (0) 0 b f (x) 1
edaytamsmtikm f (x) 1 enaH f (0) 0 .
eFIVedrIeveFobnwg x kgsmPaB f (x y ) 1f (xf)(x)ff((yy))
edaycat;Tuk y CaGefrminGaRsynwg x eK)an
2

f ' ( x )[1 f ( x )f ( y )] f ' ( x )f ( y )[f ( x ) f ( y )]


[1 f ( x )f ( y )]2
f ' ( x )[1 f 2 ( y )]
f '(x y )
[1 f ( x )f ( y )]2

f '(x y )

eroberogeday lwm pln

- TMBr 43 -

KNitviTaCMuvijBiPBelak
f ' (0)[1 f 2 ( y )]
f '(y )
[1 f (0)f ( y )]2

yk x 0 eK)an
eday f (0) 0
eK)an f '(y ) f '(0)[1 f (y )] tag f '(0)
eK)an 1 f 'f(2y()y ) eRBaH f (y ) 1
eFIVGaMgetRkaleFobnwg y elIGgTaMgBIrnsmIkarxagelIeK)an
2

f ' ( y )dy
dy y k

1 f 2 (y )

tag z f (y ) enaH dz f '(y )dy


dz
1
1
1
(
eK)an 1f '(yf 2)dy

)dz

2
2 1 z 1 z
(y )
1 z
1 dz
1 dz

2 1 z 2 1 z
1
1
ln | 1 z | ln | 1 z |
2
2
1 1 z 1 1 f (y )
ln
ln
2 1 z 2 1 f (y )

eKTaj 12 ln 11 ff ((yy )) y k
cMeBaH y 0 eK)an 12 ln 11 ff ((00)) k naM[ k 0 eRBaH f (0) 0
ehtuenH 12 ln 11 ff ((yy )) y
eroberogeday lwm pln

- TMBr 44 -

KNitviTaCMuvijBiPBelak
eKTaj)an 11 ff ((yy )) e2 y (1) b 11 ff ((yy )) e 2 y (2)
tam (1) eK)an 1 f (y ) e f (y )e
1 e2 y
naM[ f (y ) 1 e2 y .
tam (2) eK)an 1 f (y ) e f (y )e
1e 2 y
naM[ f (y ) 1 e 2 y .

2y

2y

2 y

2y

dUcenH

1 e 2x
f ( x)
1 e 2 x

eroberogeday lwm pln

1 e 2x
f ( x)
1 e 2 x

Edl f '(0) .

- TMBr 45 -

KNitviTaCMuvijBiPBelak
lMhat;TI13
cUrbgajfa
log a bc logb ca logc ab 4 (logab c log bc a log ca b )

cMeBaHRKb; a , b , c FMCagmYy .

( Malaysia National Olympiad 2010 )

dMeNaHRsay
karbgaj
tag A loga bc logb ca logc ab
nig B 4 (logab c logbc a logca b)
ehIyyk x ln a , y ln b , z ln c
cMeBaH a , b , c 1 enaH x , y , z 0
p
eK)an

tamrUbmnbreKal logq p ln
ln q
yz zx xy
1 1
1 1
1 1
x( ) y ( ) z ( )

x
y
z
y z
z x
x y
z
x
y
4x
4y
4z
B 4(

xy yz zx yz zx xy
A

eroberogeday lwm pln

- TMBr 46 -

KNitviTaCMuvijBiPBelak
tamvismPaB AB GM RKb; u , v 0 : u v 2 uv
4
1 1
4
v

b (u v )2 4uv b uuv

b
u v u v u v
eKTaj x( y1 1z ) y4xz ; y( 1z 1x ) z4yx ; z( 1x y1 ) x4zy
bUkvismPaBenHeK)an A B Bit .
dUcenH
log a bc log b ca log c ab 4 (logab c log bc a log ca b )

eroberogeday lwm pln

- TMBr 47 -

KNitviTaCMuvijBiPBelak
lMhat;TI14
eK[rgVg;pit O kaM r nigRtIekaN ABC ERbRbYlcarwkeRkArgVg; .
bnat;kat;tam O kat;RCug AB nig AC erogKaRtg; M nig N .
kMNt;TItaMgcMNuc A nigGgt; [MN] edIm,I[pRkLaRtIekaN AMN
mantmtUcbMput ?
(RbLgsisBUEkRbcaMraCFanIPMeBj 2009)

dMeNaHRsay
kMNt;TItaMgcMNuc A nigGgt; [MN]
tag S AMN CaRkLapnRtIekaN
AMN ehIy A' , B' , C'
CacMeNalEkgn O elI
RCug BC , CA , AB
erogKa .
eyIg)an
S AMN

1
1
AM .AN. sin A r ( AM AN )
2
2
AM .AN. sin A r ( AM AN )

eKTaj)an
tamvismPaB AM GM eKman AM AN 2
eroberogeday lwm pln

AM .AN

- TMBr 48 -

KNitviTaCMuvijBiPBelak
eK)an AM.AN. sin A 2r AM.AN
b AM 2 .AN2 . sin2 A 4r 2 .AM.AN
4r 2
eKTaj)an AM.AN. sin A sin A
4r 2
ehtuenH S AMN sin A
edIm,I[pRkLaRtIekaN AMN mantmtUcbMputluHRtaEt sin A 1

eKTaj)an A 90o .
kgkrNIenHeFIV[vismPaB AM AN 2 AM.AN kayCasmPaB
KW AM AN ehtuenH AMN KWCaRtIekaNsm)atkMBUl .
eday OA CaknHbnat;BuHkgnmMu BAC enaH OA MN
ehtuenH OM ON naM[ MN 2. OM
OC'
r
kgRtIekaNEkg OMC' eKman sin OMC' OM

OM
eday OMC' 45o enaH OM 2 r
eKTaj)an MN 2 2 r .
snidan
edaymMu MAN 90o enaHcMnuc M KWCaknHrgVg;manpit O nigman
viCmaRt MN 2 2 r .
eroberogeday lwm pln

- TMBr 49 -

KNitviTaCMuvijBiPBelak
lMhat;TI15
eKtag , , CargVas;mMukgRtIekaN ABC mYyEdlmanbrimaRt 2p
nigkaMrgVg;carwkeRkA R .
R2

a / cUrRsaybBaak;fa cot cot cot 3 9 . 2 1


p

b / etIeBlNaeTIbeK)ansmPaB ?
2

( Morocco National Olympiad 2011 )

dMeNaHRsay
R2

a / RsaybBaak;fa cot cot cot 3 9 . 2 1


p

eKman

1
1
1
2
2
2

cot

cot

cot

2
2
2
sin sin sin
1
1
1
27 R 2
eyIgnwgRsayfa 2 2 2 2
sin sin sin
p

tamvismPaB Cauchy Schwarz eKman


1
1
1
1 1
1
1 2

(
) (1)
2
2
2

3
sin
sin
sin
sin sin sin

edayeRbI

2
(a 1 a 2 a 3 ) 2
a12 a 2 2 a 3

b1 b 2 b 3
b1 b 2 b 3

eroberogeday lwm pln

- TMBr 50 -

KNitviTaCMuvijBiPBelak
eK)an

1
1
1
9

sin sin sin sin sin sin

tamRTwsIbTsIunUsGnuvtn_kg ABC eK)an

a
b
c
abc
2R

sin sin sin sin sin sin


eKTaj sin sin sin a b c p
2R
R
9
9R
ehtuenH 1 1 1

( 2)
sin sin sin sin sin sin
p

tam (1) & (2) eKTaj)an


1
1
1
1 81R 2 27 R 2

2
2
2
2
sin sin sin 3
p
p2

Bit

R2

dUcenH cot cot cot 3 9 . 2 1 .

p
1
1
1
b / vismPaBenHkayCasmPaBluHRtaEt

sin sin sin


2

eKTaj)an naM[ ABC CaRtIekaNsmgS .

eroberogeday lwm pln

- TMBr 51 -

KNitviTaCMuvijBiPBelak
lMhat;TI16
3 1 1
1 0 0
eK[m:aRTIs A 1 3 1 nig I 0 1 0
1 1 3
0 0 1
bgajfamansIVtcMnYnBitBIr (un ) nig ( vn ) Edlepgpat;
n IN : An un . I vn .A

EdleKnwgbBaak;tYTUeTA un nig vn

CaGnuKmn_n n .
dMeNaHRsay

bgajfa

n IN : An un . I v n .A

3 1 1
1 0 0
3 1 1
eKman A 1 3 1 0.0 1 0 1.1 3 1
1 1 3
0 0 1
1 1 3
eK)an A1 u1 .I v1 .A BitcMeBaH n 1 Edl u1 0 , v1 1 .
3 1 1 3 1 1 11 7 7
A 2 A.A 1 3 1 .1 3 1 7 11 7

1 1 3 1 1 3 7 7 11
0
0 21 7 7
10
A2 0
10
0 7 21 7 10.I 7.A

0
10 7 7 21
0
eK)an A 2 u2 .I v 2 .A BitcMeBaH n 2 .

eroberogeday lwm pln

- TMBr 52 -

KNitviTaCMuvijBiPBelak
]bmafavaBitcMeBaHRKb; n IN KW An un .I vn .A
eyIgnwgRsayfa An 1 un 1 .I vn 1 .A Bit .
eKman An 1 A.An A.(un .I vn .A) un .A.I vn .A 2
eday A.I A nig A 2 u2 .I v 2 .A
eK)an An 1 un .A vn (u 2 .I v 2 .A)
An 1 u 2 vn .I (un v 2 v n ).A
A n 1 un 1 .I v n 1 .A

Bit

Edl un 1 u 2 vn 10.vn nig vn 1 un v 2 vn un 7 vn


dUcenH n IN : An un . I vn .A
Edl (u n ) nig ( vn ) CasIVtcMnYnBitBIrkMNt;eday u1 0 , v1 1
nig u n 1 10.vn , vn 1 u n 7vn RKb; n IN .

bBaak;tYTUeTA

un

nig vn CaGnuKmn_n n

eKman un 1 10.vn , vn 1 un 7 vn RKb; n IN


tam vn 1 un 7 vn eK)an vn 2 un 1 7 vn 1
Et un 1 10vn enaH vn 2 7 vn 1 10vn
smIkarsmal;n vn 2 7 vn 1 10vn KW r 2 7r 10
b r 2 7r 10 0 manbs r1 2 , r2 5 .
tagsIVtCMnYy abn vvn 1 25vvn

eroberogeday lwm pln

n 1

- TMBr 53 -

KNitviTaCMuvijBiPBelak
an 1 vn 2 2 vn 1 7 vn 1 10 vn 2 v n 1 5an
eK)an b v 5v 7 v 10v 5v 2b
n 1
n 2
n 1
n 1
n
n 1
n
naM[ (an ) nig (bn ) CasIVtFrNImaRtmanersug 5 nig 2 erogKa .

eK)an an a1 .5n 1 nig bn b1 .2n 1 .


eday a1 v 2 2v1 7 2 5 nig b1 v 2 5v1 7 2 2
eKTaj an 5.5n 1 5n nig bn 2.2n 1 2n .
eK)anRbBn

v n 1 2 v n 5n

vn 1 5 vn 2n

dksmIkarBIrenHeK)an 3vn 5

naM[

5n 2n
vn
3

10 n
ehIy un 1 10vn 3 (5 2n )
naM[ un 103 (5n 1 2n 1 ) .
10 n 1
5n 2n
n 1
dUcenH un 3 (5 2 ) nig vn 3

eroberogeday lwm pln

- TMBr 54 -

KNitviTaCMuvijBiPBelak
lMhat;TI17
eK[ f CaGnuKmn_Cab; nig manedrIevelI IR EdlcMeBaHRKb; x, y IR
eKmanTMnak;TMng f ( x 2 y ) f (x)f (y ) nig f (x) 0 .
cUrkMNt;GnuKmn_ y f (x) .
dMeNaHRsay
kMNt;GnuKmn_ y f (x)
eKmanTMnak;TMng f ( x 2 y ) f (x)f (y )
eday f (x) 0 enaH f (y ) 0 nig f ( x 2 y ) 0
tagGnuKmn_ g(x) ln f (x) cMeBaHRKb; x IR
eK)an g( x 2 y ) ln f ( x 2 y ) eday f ( x 2 y ) f (x)f (y )

ehtuenH g( x 2 y ) ln f (x)f (y ) 12 ln f (x) ln f (y )


b g( x 2 y ) 12 g(x) g(y )
eFIVedrIeveFobnwg x kgsmPaB g( x 2 y ) 12 g(x) g(y )
edaycat;Tuk y CaGefrminGaRsynwg x eK)an
1 xy
1
xy
g' (
) g' ( x ) b g' (
) g' ( x )
2
2
2
2
eroberogeday lwm pln

- TMBr 55 -

KNitviTaCMuvijBiPBelak
yk x 0 eK)an g'( y2 ) g'(0)
eFIVGaMgetRkal g'( y2 )dy g'(o)dy g'(0)y k
tag z y2 enaH dy 2dz
eK)an 2 g'(z )dz g'(0)y k
1
k
k
2g( z ) g' (0)y k b g( z ) g' (0)y g' (0)z
2
2
2
yk a g'(0) nig b k2 eK)an g(z ) az b
b g(x) ax b eday g(x) ln f (x)
eK)an ln f (x) ax b naM[ f (x) e
dUcenH f (x) e Edl a , b IR .
ax b

ax b

eroberogeday lwm pln

- TMBr 56 -

KNitviTaCMuvijBiPBelak
lMhat;TI18
eK[ctuekaNe)a:g ABCD mYymanRCug AB a , BC b ,
CD c nig DA d .
P CacMnucmYyenAkgctuekaNehIy K , L , M , N CacMeNalEkg
ncMnuc P elIRCug [AB] , [BC] , [CD] , [DA] erogKa .
PL PM PN

h .
eKdwgfa PK
AB BC CD DA
k> cUrRsayfa h 2 22S 2 2 Edl S CapRkLan
a b c d
ctuekaN ABCD .
x>cUrbgajfa h 12
dMeNaHRsay
k>Rsayfa h 2 22S 2 2
a b c d

B
M
N

eroberogeday lwm pln

C
L

- TMBr 57 -

KNitviTaCMuvijBiPBelak
eKman

S S PAB S PBC S PCD S PDA


1
1
1
1
a.PK b.PL c.PM d.PN
2
2
2
2
PL PM PN
eday PK

h
AB BC CD DA
eKTaj PK ah , PL bh , PM ch , PN dh
eK)an S 12 a2h 12 b2h 12 c2h 12 d2h
dUcenH h 2 22S 2 2 .
a b c d
x>bgajfa h 12
eKman S S ABC SCDA 12 (ab sin B cd sin D)
eday sin B 1 nig sin D 1 enaHeK)an

1
c2 d 2
a2 b2
S (ab cd ) eday ab
; cd
2
2
2
a2 b 2 c2 d 2
eK)an S
4
tamsRmayxagelI h 2 22S 2 2
a b c d
a2 b 2 c2 d 2
2(
) 1
dUcenH h 2 2 4 2 2 2 .
a b c d

eroberogeday lwm pln

- TMBr 58 -

KNitviTaCMuvijBiPBelak
lMhat;TI19

eK[ctuekaNe)a:g ABCD mYymanRCug AB a , BC b


CD c nig DA d . O CacMnucmYyenAkgctuekaNenHEdl
OAB OBC OCD ODA 0 90o )
k> cUrRsayfa tan a2 b24Sc2 d2
Edl S CapRkLanctuekaN ABCD .
x> kMnt; edaydwgfa tan mantmFMbMput .
dMeNaHRsay

k> Rsayfa tan a2 b24Sc2 d2


D
C
O

eroberogeday lwm pln

- TMBr 59 -

KNitviTaCMuvijBiPBelak

tag OA x , OB y , OC z , OD t .
tamRTwsIbTkUsIunUskg OAB , OBC , OCD , ODA
y 2 a 2 x 2 2ax cos

(1)

z 2 y 2 b 2 2by cos

( 2)

t 2 z 2 c 2 2zc cos

( 3)

x 2 t 2 d 2 2td cos

( 4)

bUksmIkar (1) , (2) , (3) & (4) eKTaj)an


a2 b 2 c2 d 2
( 5)
cos
2(ax by cz dt )

m:ageTot S SOAB SOBC SOCD SODA


b S 12 (ax by cz dt ) sin
eKTaj sin ax by2S cz dt (6)
EckTMnak;TMng (6) nig (5) Gg nig GgeK)an
4S
.
tan 2
2
2
2
a b c d
x> kMnt; edaydwgfa tan mantmFMbMput
eKman S S ABC SCDA 12 (ab sin B cd sin D)
eday sin B 1 nig sin D 1 enaHeK)an
eroberogeday lwm pln

- TMBr 60 -

KNitviTaCMuvijBiPBelak
a2 b2
c2 d 2
1
; cd
S (ab cd ) eday ab
2
2
2
a2 b 2 c2 d 2
eK)an S
4
eday tan 2 24S 2 2
a b c d
a2 b 2 c2 d 2

1
tan
a2 b 2 c2 d 2

eKTaj
dUcenHedIm,I[ tan mantmFMbMputluHRtaEt 45o .

eroberogeday lwm pln

- TMBr 61 -

KNitviTaCMuvijBiPBelak
lMhat;TI20
cMeBaHRKb; x ; y 0 cUrRsaybBaak;vismPaB
x 2 x 1 y 2 y 1 x 2 x 1 y 2 y 1 2( x y )

(Kazakhstan NMO 2010)

dMeNaHRsay
RsaybBaak;vismPaB
x 2 x 1 y 2 y 1 x 2 x 1 y 2 y 1 2( x y )

eRCIserIsctuekaNe)a:g ABCD mYymanGgt;RTUg AC nig BD RbsBV


eroberogeday lwm pln

- TMBr 62 -

KNitviTaCMuvijBiPBelak
KaRtg;cMnuc I Edl IA x

, IC y , IB ID 1

nig AIB CID 60o . tamRTwsIbTkUsIunUseK)an


AB x 2 x 1 , CD y 2 y 1
AD x 2 x 1 , BC y 2 y 1

tamRTwsIbTGWEl AC.BD AB.CD AD.BC


eday AC AI IB x y

, BC BI IC 2

dUcenH
x 2 x 1 y 2 y 1 x 2 x 1 y 2 y 1 2( x y )

eroberogeday lwm pln

- TMBr 63 -

KNitviTaCMuvijBiPBelak
lMhat;TI21
eK[ a , b , c CacMnYnBitviCman . cUrRsaybBaak;fa
( 2a b c)2
( 2b c a )2
( 2c a b )2
2
2
8
2
2
2
2
2a (b c)
2b (c a )
2c (a b )
(USAMO 2003)

dMeNaHRsay
2
2
2
(
2
a

c
)
(
2
b

a
)
(
2
c

b
)
RsaybBaak; 2
2
2
8
2
2
2
2a (b c)

2b (c a )

2c (a b )

rebobTI1
tag

( 2a b c)2
( 2b c a )2
( 2c a b )2
T 2
2
2
2
2
2a (b c)
2b (c a )
2c (a b )2

yk s a b c enaHkenSam T Gacsresr
(a s )2
(b s ) 2
(c s ) 2
T 2
2
2
2
2
2a (a s )
2b (b s )
2c (c s )2

eKman

(a s )2
a 2 2as s 2
2
2
2
2a (a s )
3a 2as s 2
1 3a 2 6as 3s 2
. 2
3 3a 2as s 2
1
8as 2s 2
(1 2
)
2
3
3a 2as s

eroberogeday lwm pln

- TMBr 64 -

KNitviTaCMuvijBiPBelak
(a s )2
1
8as 2s 2
1
8as 2s 2
2

2
2
2
3 9a 6as 3s
3 ( 3a s )2 2s 2
2a (a s )

eday (3a s)2 0 enaH (3a s)2 2s2 2s2


8as 2s 2
8as 2s 2
4a
4a
1
1
b

2
2
2
s
abc
( 3a s ) 2s

eKTaj

2s

(a s )2
4
4a
(1)

2
2
3 abc
2a (a s )

RsaybMPWdUcKaxagelIEdreK)an
(b s )2
4
4b

( 2)
2
2

3
a
b
c
2b (b s )
(c s )2
4
4c

( 3)
2
2
3 abc
2c (c s )

bUkvismPaB (1) , (2) nig (3) eK)an

4 4 4 4a 4b 4c
44 8

3 3 3
abc
( 2a b c)2
( 2b c a )2
( 2c a b )2
dUcenH 2
2
2
8
2
2
2
2a (b c)
2b (c a )
2c (a b )
T

rebobTI2
tag

( 2a b c)2
( 2b c a )2
( 2c a b )2
T 2
2
2
2
2
2a (b c)
2b (c a )
2c (a b )2

yk x a b , y b c , z c a
eroberogeday lwm pln

- TMBr 65 -

KNitviTaCMuvijBiPBelak
eK)an

x z 2a b c

x y 2b c a
z y 2c a b

kenSam T GacsresrCa

nig

2a x z y

2b x y z
2c z y x

2( x z )2
2( z y )2
2( y x )2
T

2
2
2
2
( x z y ) 2y
( z y x ) 2x
( y x z )2 2z 2

tamvismPaB Cauchy Schwarz 2(u2 v 2 ) (u v )2


eK)an 2(x z y )2 2y 2 (x z y y )2 (x z )2
2( x z y )2 4y 2 ( x z )2 2y 2
1
( x z y )2 2y 2 ( x z )2 y 2
2
2( x z )2
2( x z )2
4

eKTaj
( x z y )2 2y 2 1 ( x z )2 y 2
2y 2
1
2
( x z )2
2y 2
y2
2
2
2
eday (x z ) 2(x z ) naM;[
2
2
(x z)
x z2

2y 2
x2 y 2 z 2
1

2
(x z )
x2 z 2

eKTaj)an

2( x z )2
4( x 2 z 2 )
(1)
2
2
2
2
2
( x z y ) 2y
x y z

eroberogeday lwm pln

- TMBr 66 -

KNitviTaCMuvijBiPBelak
RsaybMPWdUcxagelIEdreK)an
2( z y )2
4( z 2 y 2 )
2
( 2)
2
2
2
2
( z y x ) 2x
x y z
2( y x )2
4( y 2 x 2 )
2
( 3)
2
2
2
2
( y x z ) 2z
x y z

bUkvismPaB (1) , (2) , (3) eK)an


4( x 2 z 2 ) 4( z 2 y 2 ) 4( y 2 x 2 )
T
8
2
2
2
x y z
( 2a b c)2
( 2b c a )2
( 2c a b )2
dUcenH 2
2
2
8
2
2
2
2a (b c)
2b (c a )
2c (a b )

rebobTI3
tag

( 2a b c)2
( 2b c a )2
( 2c a b )2
T 2
2
2
2
2
2a (b c)
2b (c a )
2c (a b )2

eKmansmPaB (2u v )2 2(u v )2 3(2u2 v 2 )


edayyk u a nig v b c enaHeK)an

( 2a b c)2 2(a b c)2 3 2a 2 (b c)2

b (2a b c)2 3 2a2 (b c)2 2(a b c)2


EckGgTaMgBIrnwg 2a2 (b c)2 eK)an
( 2a b c)2
2(a b c)2
eyIg)an 2
3 2
2
2
2a (b c)

eroberogeday lwm pln

2a (b c)

- TMBr 67 -

KNitviTaCMuvijBiPBelak
kenSam T GacbMElgCa
(a b c )2
(b a c ) 2
(c a b )2
2
2
T 92 2
2
2
2

2
a
(
b
c
)
2
b
(
c
a
)
2
c
(
a
b
)

edIm,IRsayfa T 8 eyIgRtUvRsay[eXIjfa
(a b c )2
(b a c ) 2
(c a b )2
1

S 2
2a (b c)2 2b 2 (c a )2 2c2 (a b )2 2

eKman 2a2 (b c)2 2a2 b2 c2 2bc


eday 2bc b2 c2 enaH 2a2 (b c)2 2(a2 b2 c2 )
RsaydUcKaEdr 2b2 (c a)2 2(a2 b2 c2 )
ehIy 2c2 (a b)2 2(a2 b2 c2 )
(a b c )2 (b a c )2 (c a b ) 2
eKTaj S
2
2
2
2(a b c )

edaykenSam (a b c)2 (b a c)2 (c a b)2


esInwg 3(a2 b2 c2 ) 2(ab bc ca)
enaH S 32 ab2 bc2 ca2

a b c
a2 b 2 b 2 c2 c2 a2
eKman 2 2 2 ab bc ca
b a2 b2 c2 ab bc ca naM[ S 32 1 12 Bit .

eroberogeday lwm pln

- TMBr 68 -

KNitviTaCMuvijBiPBelak
rebobTI4
( 2a b c)2
( 2b c a )2
( 2c a b )2
Rsayfa 2
2
2
8
2
2
2
2a (b c)
2b (c a )
2c (a b )
bc
ca
ab
x
,
y
,
z
tag a

b
c

Edl

1
1
1
a
b
c

1
x1 y 1 z1 abc abc abc

vismPaBsmmUl

( 2 x )2 ( 2 y )2 ( 2 z )2

8
2
2
2
2 x
2 y
2 z

( 2 u )2 8
1
1
eyIgnwgRsayfa

RKb;
u0
2
3 1 u 3
2u
( 2 u )2 8
1
1
(u 5)(u 2)2
eKman


0 Bit
2
2
3
1

u
3
2u
3( 2 u )(1 u )
( 2 u )2
1
1 8

(*)
2
1

u
3
3
2u

ehtuenH
tam (*) eK)an

( 2 x )2 ( 2 y )2 ( 2 z )2
1
1
1

1 8
2
2
2
1

x
1

y
1

z
2 x
2 y
2 z

dUcenH

( 2a b c)2
( 2b c a )2
( 2c a b )2
2
2
8
2
2
2
2
2a (b c)
2b (c a )
2c (a b )

eroberogeday lwm pln

- TMBr 69 -

KNitviTaCMuvijBiPBelak
lMhat;TI22
cUrbgajfa 1 1 1 1 1 1

1
1

a b c d
ac bd
cMeBaHRKb;cMnYnBitviCman a , b , c , d .

dMeNaHRsay
eKman
X

nig

1
1 1

a b
Y

1
1 1

c d
1

evotNam

1962

ab
cd
abc abd acd bcd

ab cd
(a b )(c d )

(a c)(b d )
abcd

1
1

ac bd
(a c)(b d ) abc abd acd bcd

YX
abcd
(a b )(c d )

eK)an
bnab;BItRmUvPaKrYm rYcsRmYleK)an

(ad bc )2
YX
0 naM[ Y X
(a b )(c d )(a b c d )
dUcenH 1 1 1 1 1 1 1 1 1 .

a b c d
ac bd

eroberogeday lwm pln

- TMBr 70 -

KNitviTaCMuvijBiPBelak
lMhat;TI23
cUrbgajfa

1
1
1
4

( x y )2 ( y z )2 ( z x )2 xy yz zx

cMeBaHRKb;cMnYnBitminGviCmanxusKa x , y , z .
dMeNaHRsay
bgajfa 1

( x y )2

evotNam

2008

1
1
4

( y z )2 ( z x )2 xy yz zx

eday x , y , z CacMnYnBitxusKa nig minGviCmanenaHeKGacsntyk


zyx0 .
tag A 1 2 1 2 1 2
(x y )

(y z )

(z x)

nig B xy yz zx .
yk y x p , z x p q Edl p 0 , q 0
eK)an A 12 12 1 2
p

(p q )

nig B x(x p) (x p)(x p q) x(x p q)


3x 2 2( 2p q )x p 2 pq

edaysarEt x 0 enaH B p2 pq p(p q)


eroberogeday lwm pln

- TMBr 71 -

KNitviTaCMuvijBiPBelak
eK)an

1
1
1
AB 2 2
p(p q )
2
q
(p q )
p
p q p( p q )
p

p
pq
q2
1

q p(p q )
q
1

pq
p
q2

q
q
p( p q )
2(
)
p pq
q2
q2
p(p q )
2

p(p q )
q2
q2
p( p q )
q2
eday p(p q) 2 2 p(p q) . p(p 2 q) 2
q
q
eKTaj)an A B 2 2 4 naM[ A B4
dUcenH 1 2 1 2 1 2 xy yz4 zx .
(x y )
(y z )
(z x)
3x 2 2( 2p q )x 0
vismPaBenHkayCasmPaBluHRtaEt
p(p q ) q 2

eKTaj)an x 0 nig p(p q) q2 ehIy y p , z p q


enaH z y q tam p(p q) q2 eKTaj)an yz (z y )2
naM[ z 2 3yz y 2 0 eday z y enaH z 3 5 1
y

eroberogeday lwm pln

- TMBr 72 -

KNitviTaCMuvijBiPBelak
lMhat;TI24
cUrkMNt;tYTUeTAnsIVtEdlkMNt;eday
x 3 , x 4 nig x x nx cMeBaHRKb; n .
dMeNaHRsay
KNnatYtUeTA x
eKman
0

2
n 1

n 1

x0 3 0 3
x1 4 1 3

x 2 x 0 2 x1 9 4 5 2 3

]bmafa x n 2 , x n 3 Bit
eyIgnwgRsayfa x n 4
eKman x x nx
n 1

n 1

n 1

2
n 1

(n 2)2 n(n 3)

dUcenH

n 2 4n 4 n 2 3n
n4
xn n 3 .

eroberogeday lwm pln

- TMBr 73 -

KNitviTaCMuvijBiPBelak
lMhat;TI25
7
.
cUrkMNt;RKb;cMnYnBit x ebIeKdwgfa 128 27
18
6
dMeNaHRsay
kMNt;cMnYnBit x
7
eKman 128 27
tag
a 2 nig b 3 Edl a 0,b 0

18
6
smIkarkayCa
x

a3 b3
7

a 2b ab 2 6
(a b)(a 2 ab b 2 ) 7

ab(a b)
6
a 2 ab b 2 7

ab
6
6a 2 6ab 6b 2 7ab
6a 2 13ab 6b 2 0
(2a 3b)(3a 2b) 0

b ba 23 32
eKTaj)an

dUcenH x 1 , x 1 .
a 2

b 3

eroberogeday lwm pln

eday

a 2

b 3

- TMBr 74 -

KNitviTaCMuvijBiPBelak
lMhat;TI26
cUrKNnaplbUk
Sn

3
4
n2

...
1! 2! 3! 2! 3! 4!
n! (n 1)! (n 2)!

dMeNaHRsay
KNnaplbUk S
2
eKman S k ! (k k 1)!
(k )!
n

k 1

k2
k ! (k 1)! (k 2)!
k2

k ![1 (k 1) (k 1)(k 2)]


k2

k !(1 k 1 k 2 3k 2)
k2
1

k !(k 2)2 k !(k 2)


k 1
(k 2) 1

(k 2)!
(k 2)!
1
1

(k 1)! (k 2)!
dUcenH Sn kn1 ak 12 (n 1 2) ! .

tag

ak

eroberogeday lwm pln

- TMBr 75 -

KNitviTaCMuvijBiPBelak
lMhat;TI27 ( China 1983 )
eK[ f CaGnuKmn_kMNt;elIcenaH [0 ,1] edaydwgfa
f (0) f (1) 1 nig | f (a) f (b) | | a b |
cMeBaHRKb; a b kgcenaH [0 ,1] .
cUrbgajfa | f (a) f (b) | 12 .
dMeNaHRsay
bgajfa | f (a) f (b) | 12
-krNITI1
cMeBaH | a b | 12 enaHeK)an | f (a) f (b) | | a b | 12 Bit
-krNITI2
cMeBaH | a b | 12 enaHtamlkNqHeKGacsntfa a b
eKman | f (a) f (b) || f (a) f (1) f (0) f (b) |
tamvismPaBRtIekaNeK)an
| f (a) f (b) | | f (a) f (1) | | f (0) f (b) |

| f (a) f (b) | | a 1 | | 0 b | 1 a b 1 (a b)

1
2

dUcenH | f (a) f (b) | 12 .


eroberogeday lwm pln

- TMBr 76 -

KNitviTaCMuvijBiPBelak
lMhat;TI28 ( AIME 1988 )
cUrkMNt;KUncMnYnKt; (a ,b) edaydwgfa ax bx 1
Eckdac;nwg x x 1 .
dMeNaHRsay
kMNt;KUncMnYnKt; (a ,b)
tag p nig q Cabsrbs;smIkar x x 1 0 enaHtamRTwsIbTEvt
eK)an p q 1 nig pq 1 .
edIm,I[ ax bx 1 Eckdac;nwg x x 1 luHRtaEt p nig q
Cabsrbs;smIkar ax bx 1 0 Edr .
eK)an ap bp 1 nig aq bq 1
eKTaj)an q (ap bp ) q
b ap b q eRBaH pq 1
ehIy p (aq bq ) p b aq b p
ehtuenH (ap b) (aq b) p q
eKTaj)an a p p qq (p q)(p q )(p q )(p q )
eday p q 1 enaH p q (p q) 2pq 1 2 3
17

16

17

16

17

17

16

16

16

17

17

16

16

16

16

16

17

16

16

16

16

16

16

16

p 4 q 4 (p 2 q 2 )2 2p 2q 2 9 2 7
p 8 q 8 (p 4 q 4 )2 2p 4q 4 49 2 47

eroberogeday lwm pln

- TMBr 77 -

KNitviTaCMuvijBiPBelak
dUcenH a 1.3.7.47 987 .
ma:geTottam ap bp 1 nig aq bq
eK)an ap bp aq bq
eKTaj)an b pp qq .a Et a p p qq
17

17

enaH

16

16

17

17

17

17

16

16

16

16

16

16

p17 q17
(p16 p15q p14q 2 ... q16 )
b
pq
[(p16 q16 ) pq(p14 q14 ) p 2q 2 (p12 q12 ) ....
... p 7q 7 (p 2 q 2 ) p 8q 8 ]
[(p16 q16 ) (p14 q14 ) ... (p 2 q 2 ) 1]

tag S p q RKb; n 1 enaH S


eKman S p q
2n

2n

2n

2n 4

3,S 4 7 ,S 8 47

2n 4

2n 4

(p 2 q 2 )(p 2n 2 q 2n 2 ) p 2q 2 (p 2n q 2n )
3S 2n 2 S n

yktm n 3 ,4,5,6,7 ,8 CMnYskg S 3S S


eK)an S 18 ,S 47 ,S 123 ,S 322 , S 843
nig S 2207 .
eK)an b (2207 843 322 123 47 18 7 3 1)
b b 1597 .
dUcenH (a,b) (987 , 1597) .
2n 4

10

12

2n 2

2n

14

16

eroberogeday lwm pln

- TMBr 78 -

KNitviTaCMuvijBiPBelak
lMhat;TI29
edaHRsaykgsMNMucMnYnBitnsmIkar
x 3x x 2 .
3

dMeNaHRsay
edaHRsaysmIkar x 3x x 2
smIkarmannyluHRtaEt x 2 0 b x 2 .
-cMeBaH x 2 eKman x 4x x(x 2) 0 (i )
ehIy x x 2 (x 1)(x 2) 0 b x x 2 (ii)
bUkvismPaB (i ) & (ii) eK)an x 3x x 2
dUcenHsmIkar x 3x x 2 KanbscMeBaH x 2 .
-cMeBaH 2 x 2 eyIgGactag x 2 cos a Edl 0 a
smIkarGacsresrCa 8cos a 6cosa 2cosa 2
3

2(4cos 3 3cosa) 4cos 2

a
2

a
cos 3a | cos |
2

eday 0 a enaH cos a2 0


smIkarsmmUl cos 3a cos a2
eroberogeday lwm pln

- TMBr 79 -

KNitviTaCMuvijBiPBelak
a
a
2k 1 , 3a 2k 2
2
2
b a 4k51 , a 4k72 (k 1 ,k 2 )
eday 0 a enaHeKTaj)an a { 0 , 45 , 47 }
dUcenH x 2cos 0 2 , x 2cos 45 , x 2cos 47

eKTajbs

3a

eroberogeday lwm pln

- TMBr 80 -

KNitviTaCMuvijBiPBelak
lMhat;TI30 ( Korean Mathematics Competition 2000 )
cUrkMNt;RKb;cMnYnBit x Edlepgpat;smIkar
2x 3x 4x 6x 9x 1

dMeNaHRsay
kMNt;RKb;cMnYnBit x

2x 3x 4x 6x 9x 1
tag a 2x 0 nig b 3x 0

smIkarkaCa

a b a 2 ab b 2 1
1 a 2 b 2 a b ab 0
2 2a 2 2b 2 2a 2b 2ab 0
(1 2a a 2 ) (1 2b b 2 ) (a 2 2ab b 2 ) 0
(1 a)2 (1 b)2 (a b)2 0

eKTaj)an a b 1
ehtuenH 2 3 1 naM[ x 0 .
dUcenHsmIkarmanbsEtmYyKt;KW x 0 .
x

eroberogeday lwm pln

- TMBr 81 -

KNitviTaCMuvijBiPBelak
lMhat;TI31 ( China 1992 )
cUrbgajfa 16 1 17
k
dMeNaHRsay
bgajfa 16 1 17
80

k 1

80

k 1

eKman 2(

k 1 k)

2
2
1

k 1 k 2 k
k

80
1
2 ( k 1 k ) 16
k 1
k 1
k
2
2
1
ehIy 2( k k 1)

k k 1 2 k
k
eK)an k801 1 1 2k80 2 ( k k 1) 2 80 1 17
k
dUcenH 16 k801 1 17 .
k

eK)an

80

eroberogeday lwm pln

- TMBr 82 -

KNitviTaCMuvijBiPBelak
lMhat;TI32
z1 z 2 z 2 2
eK[ z1 , z 2 , z 3 CacMnYnkMupicEdl z12 z 22 z 32 3
z z z 4
1 2 3
cUrKNnatm S z z 1z 1 z z 1z 1 z z 1z 1
1 2
3
2 3
1
3 1
2

dMeNaHRsay
KNnatm S
eKman z z z
1

1 z 1z 2 z 3 1 (z 1 z 2 z 3 )
z 1z 2 1 z 1 z 2 (z 1 1)(z 2 1)

RsaybMPWdUcKaenHEdreK)an
z 2 z 3 z1 1 (z 2 1)(z 3 1), z 3 z1 z 2 1 (z1 1)(z 3 1)
z1 z 2 z 3 3
1
S

cyc (z 1)(z 1)
(z 1 1)(z 2 1)(z 3 1)
1
2

23
z1z 2 z 3 (z 1z 2 z 2 z 3 z 3 z 1 ) z1 z 2 z 3 1

2
2
2

10 2(z1z 2 z 2 z 3 z 3 z1 ) 10 (4 3)
9

eRBaH 2(z z
1

z 2 z 3 z 3 z 1 ) (z 1 z 2 z 3 )2 (z 1 2 z 2 2 z 3 2 )

eroberogeday lwm pln

- TMBr 83 -

KNitviTaCMuvijBiPBelak
lMhat;TI33
eK[ x , y , z CabIcMnYnBitviCmanEdl x y z 1 .
cUrkMNt;tmtUcbMputn 1 x x 1 y y 1 z z .
dMeNaHRsay
kMNt;tmtUcbMputn S 1 x x 1 y y 1 z z
eday x , y , z 0 nig x y z 1 enaHeKTaj 0 x, y, z 1
cMeBaH 0 t 1 tag f (t) t(1 t )
eK)an [f (t)] t (1 t ) b 8 f (t) 8t (1 t )
tamvismPaB AM GM eK)an
4

8 8

8 8

8t 8 8 8t 8
8
8
8
8
8
8
8t (1 t ) 8t .(1 t )(1 t )...(1 t )

9
8

9
9
8
8
eKTaj 8 f (t) 9 b f (t) 4 8 9 Et f (t) t(1 t 8 )

3
4
1
1
39
enaHeKTaj)anfa t(1 t 8 ) f (t) 8 (*)
x3
y3
z3
eKman S 1 x8 1 y 8 1 z 8

eroberogeday lwm pln

- TMBr 84 -

KNitviTaCMuvijBiPBelak
x4
y4
z4
eKGacsresr S x(1 x8 ) y(1 y 8 ) z(1 z 8 )

tamvismPaB (*) eKTaj)an


4
39 4
39
4
4
S
(x y z )
eRBaH
x4 y 4 z 4 1
8
8
x3
y3
z3
dUcenHtmtUcbMputn S 1 x8 1 y 8 1 z 8
4

KW

39 9 4 3

8
8
4

Smin

eroberogeday lwm pln

- TMBr 85 -

KNitviTaCMuvijBiPBelak
lMhat;TI34 ( Iran 1996 )
eK[bIcMnYnBitminGviCman a , b , c nigminsUnRBmKaBIr .
cUrRsaybBaak;fa

1
1
1 9

(ab bc ca)
2
2
2
(a b) (b c) (c a) 4

dMeNaHRsay
RsaybBaak;fa

1
1
1 9
(ab bc ca)
(*)

2
2
2
(b c )
(c a ) 4
(a b )
tag x a b c , y ab bc ca , z abc

eKmansmPaB
(a b )(b c)(c a ) (a b c)(ab bc ca ) abc

xy z

nig (a b)2 (a c)2 (x2 y )2 4x(xy z )


cyc

( x 2 y )2 4x( xy z ) 9
vismPaB (*) xagelIsmmUl y

2
( xy z )

4
smmUl 4x4y 17x2y 2 4y 3 34xyz 9z 2 0

xy(x3 4xy 9z) y(x4 5x2y 4y2 6xz) z(xy 9z) 0 (**)

eroberogeday lwm pln

- TMBr 86 -

KNitviTaCMuvijBiPBelak
tamvismPaB Schur cMeBaHRKb;cMnYnBitminGviCman x , y , z eKman
x( x y )( x z ) 0 x3 4xy 9z 0

(1)

cyc

x 2 ( x y )( x z ) 0 x4 5x2y 4y 2 6xz 0 ( 2)
cyc

tamvismPaB AM GM eKman
(a b c)(ab bc ca ) 9abc xy 9z 0 ( 3)

tam (1), (2) & ( 3) eKTaj)an (**) Bit .


dUcenH (ab bc ca) 1 2 1
(a b )

eroberogeday lwm pln

(b c )

1 9

2
(c a ) 4

- TMBr 87 -

KNitviTaCMuvijBiPBelak
lMhat;TI35
eK[ a , b , c CabIcMnYnBitminGviCman .
cUrRsayfa a3 b3 c3 3abc 2 ( b 2 c a)3 .
dMeNaHRsay
tamvismPaB AM GM eKman a3 b3 c3 3abc
enaH a3 b3 c3 3abc 0 .
ebI b 2 c a 0 enaHvismPaBxagelIBitCanic .
eyIg]bmafa b 2 c a 0 .

tag T a3 b3 c3 3abc 2( b 2 c a)3


yk b a 2x nig c a 2y enaH b c 2a 2x 2y
Edl x 0 , y 0 . eK)an
T a3 (a 2x)3 (a 2y)3 3a(a 2x)(a 2y) 2(x y)3

bnab;BIbRgYmrYceK)an
T 12a( x 2 xy y 2 ) 6( x y )( x y )2 6( x y )( x y )2
bc
bc 2
T 6(
a )(
) 0 Bit
2
2
dUcenH a3 b3 c3 3abc 2 ( b 2 c a)3 .

eroberogeday lwm pln

- TMBr 88 -

KNitviTaCMuvijBiPBelak
lMhat;TI36
eK[RtIekaN ABC mYymanRCug BC a , CA b , AB c
ehIymMukg A , B , C CamMuRsYcbmMuEkg .
tag S CapRkLan ABC
cUrRsaybBaak;fa 14 14 14 9 2 .
a

dMeNaHRsay
RsaybBaak;fa 14
a

16 S

1
1
9

b 4 c4 16 S 2

eKman S2 p(p a)(p b)(p c) Edl p a b2 c


tag x b2 c2 a2 , y c2 a2 b2 , z a2 b2 c2
Edl x , y , z 0 nig minGacmanBIrsUnRBmKa .
eK)an x y 2c2 , y z 2a2 , z x 2b2
ehIy 16S2 (a b c)(b c a)(c a b)(a b c)
4b 2c2 (b 2 c2 a 2 )2
( x y )( z x ) x 2 xy yz zx
vismPaB 14 14 14 9 2 GacbEmgeTACa
a
b
c
16 S
4
4
4
9

( x y )2 ( y z )2 ( z x )2 xy yz zx

eroberogeday lwm pln

- TMBr 89 -

KNitviTaCMuvijBiPBelak
( xy yz zx ) [

1
1
1
9

]
4
( x y )2 ( y z )2 ( z x )2

tamlkNHGUm:UEsneKGaceRCIserIsyk xy yz zx 1 enaHivsmPaB
xagelIsmmUl 4 (x y )2 (x z )2 9(x y )2 (y z )2 (x z )2
cyc

eday (x y )(x z ) x2 xy xz yz x2 1
ehIy (x y )(y z )(x z ) (x2 1)(y z )
x2 (y z ) y z
x( xy xz) y z
x(1 yz ) y z
x y z xyz

eK)an 4 (x2 1)2 9(x y z xyz)2


cyc

4[( x 2 1)2 ( y 2 1)2 ( z 2 1)2 ] 9( x y z xyz )2

4[(x4 y 4 z 4 ) 2( x2 y 2 z 2 ) 3] 9(x y z xyz)2 (*)

tag S x y z nig P xyz


Edl S a2 b2 c2 0 nig P 0 eRBaH x , y , z 0 .
eK)an S2 (x y z )2 x2 y 2 z 2 2
ehIy (S2 2)2 x4 y 4 z 4 2(x2y 2 y 2z 2 y 2z 2 )
eday xy yz xz 1 enaH (xy yz xz)2 1
eroberogeday lwm pln

- TMBr 90 -

KNitviTaCMuvijBiPBelak
b x2y 2 y 2z 2 x2z 2 2xyz(x y z ) 1
b x2y 2 y 2z 2 x2z 2 1 2S P
enaH (S2 2)2 x4 y 4 z 4 2(1 2S P)
naM[ x4 y 4 z 4 S4 4S2 4SP 2
vismPaB (*) xagelIsmmUl
4(S 4 4S 2 4SP 2 2S 2 4 3) 9(S P )2
4(S 4 2S 2 4SP 1) 9(S P )2
4S 4 8S 2 16SP 4 9(S P )2
9S 2 (4S 2 1)(S 2 4) 16SP 9(S P )2 (**)

-cMeBaH S 2 eKman (4S2 1)(S2 4) 16SP 0


naM[vismPaB (**) Bit
eRBaH 9S2 (4S2 1)(S2 4) 16SP 9S2 9(S P )2 .
-cMeBaH 0 S 2 vismPaB (**) Gacsresr
9S 2 ( 4S 2 1)(S 2 4) 16SP 9S 2 18SP 9P 2
(4S 2 1)(S 2 4) 34SP 9P 2 0

tag T (4S2 1)(S2 4) 34SP 9P 2


eyIgnwgRsayfa T 0 .
eKman (x y z )(xy yz xz) 9xyz tam AM GM
eday xy yz zx 1 enaH S 9P
eroberogeday lwm pln

- TMBr 91 -

KNitviTaCMuvijBiPBelak
ehIy T (4S2 1)(S2 4) P(S 9P ) 33SP
eK)an T (4S2 1)(S2 4) 33SP
tamvismPaB Schur eKman x2 (x y )(x z ) 0
cyc

eday x2 (x y )(x z ) x4 x2yz x3 ( y z )


eK)an ( x4 ) xyz ( x) x3 (y z )
cyc

cyc

cyc

eday (x4 ) S4 4S2 4SP 2

, xyz x SP

cyx

cyc

nig x3 (y z ) x2 (xy xz) x2 (1 yz )


cyc

cyc

cyc
2

x 2 xyz x S 2 SP
cyc

cyc

eK)an S4 4S2 5SP 2 S2 SP 2


6SP S 4 5S 2 4

ehtuenH

6SP (4 S 2 )(S 2 1)
11
T (4S 2 1)(S 2 4) 6SP
2
11
T (4S 2 1)(S 2 4) (4 S 2 )(S 2 1)
2
3
T (4 S 2 )(S 2 3)
2

eroberogeday lwm pln

- TMBr 92 -

KNitviTaCMuvijBiPBelak
eday 0 S 2 enaH 4 S2 0
ehIy S2 (x y z )2 3( xy yz zx) 3 enaH S2 3 0
eKTaj)an T 32 (4 S2 )(S2 3) 0 Bit .
srubmkeK)an (xy yz zx) [
dUcenH

1
1
1
9

a4 b4 c4 16 S 2

eroberogeday lwm pln

1
9
1
1
]

4
( x y )2 ( y z )2 ( z x )2

- TMBr 93 -

KNitviTaCMuvijBiPBelak
lMhat;TI37 Turkey 2007
eK[bIcMnYnBitviCman a , b , c Edl a b c 1 . cUrRsaybBaak;fa
1
1
1
1

ab 2c 2 2c bc 2a 2 2a ca 2b 2 2b ab bc ca

dMeNaHRsay
CadMbUgeyIgnwgRsayfa

1
ab

ab 2c 2 2c (ab bc ca)2

smmUl (ab bc ca)2 ab(ab 2c2 2c)


smmUl b2c2 c2a2 2abc(a b c) 2abc2 2abc
eday a b c 1 enaH b2c2 c2a2 2abc 2abc2 2abc
smmUl b2c2 c2a2 2abc2 c2 (a b)2 0 Bit
1
ab

ehtuenH
(1)
2
2
ab 2c 2c (ab bc ca)
1
bc

( 2)
2
2
bc 2a 2a (ab bc ca )
1
ca

( 3)
2
2
ca 2b 2b (ab bc ca )

bUkvismPaB (1) , (2) & (3) eK)an


1
1
1
1

ab 2c 2 2c bc 2a 2 2a ca 2b 2 2b ab bc ca

eroberogeday lwm pln

- TMBr 94 -

KNitviTaCMuvijBiPBelak
lMhat;TI38
eK[ a , b , c CabIcMnYnBitviCman . cUrRsaybBaak;fa
a2
2a 2 ab b 2

dMeNaHRsay

b2
2b 2 bc c2

eyIgBinitGnuKmn_ f (x)

x2

c2
2c2 ca a 2

abc
2

RKb; x 0

2x x 1
vismPaBxagelIsmmUl bf ( ba ) cf ( bc ) af ( ac ) a b2 c
2

.
edIm,IRsaybBaak;vismPaBenHeyIgRtUvkMNt;GnuKmn_lIenEGmYyEdl
f ( x ) x (*) ehIy nig CaBIrcMnYnBitEdlbMeBjlkxN (*)
BitcMeBaHRKb; x 0 .
edaysaEtvismPaBBitcMeBaH a b c enaHeyIgnwgkMNt;rk nig
EdleFIV[ExSekag (c) : y f (x) b:Hnwg (d) : y x Rtg; x 1
eBalKWRtUv[ f (1) nig f ' (1) .
eday f (1) 12 enaH 12
eKman f ' (x)

2x 2x 2 2x 1

eroberogeday lwm pln

4x 1

2 2x 2 2x 1
2x 2 2x 1

.x 2

- TMBr 95 -

KNitviTaCMuvijBiPBelak
5
11
11
eK)an f ' (1) 4 4 16
enaH 16
ehIy 12 163
11x 3
x2
11x 3

ehtuenH f (x) 16 b 2
(**)
16
2x x 1
-ebI 0 x 113 enaHvismPaB (**) BitCaniceRBaHGgTI1CakenSamviCman
CanicRKb; x 0 ehIyGgTIBIrGviCman .
-ebI x 113 enaHvismPaB (**) Gacsresr
2
2
2
2
2

x
(
11
x
3
)
(
x

1
)
(
14
x
39x 9)

0
2

2
6
25
256( 2x x 1)
2x 2x 1
vaBitcMeBaHRKb; x 113 eRBaH 14x2 39x 9 0 .
tam (**) eKCMnYs x eday ba , bc , ac eK)an
a
b
c 11a 3b 11b 3c 11c 3a
bf ( ) cf ( ) af ( )
b
c
a
16
8a 8b 8c a b c

Bi
t
16
2
4

dUcenH

a2
2a 2 ab b 2

b2
2b 2 bc c2

eroberogeday lwm pln

c2
2c2 ca a 2

abc
2

- TMBr 96 -

KNitviTaCMuvijBiPBelak
lMhat;TI39
eK[ a , b , c CabIcMnYnBitxusKa .
a2
b2
cUrRsayfa

2
2
(b c )

(c a )

c2
2
2
(a b )

dMeNaHRsay
eKman x2 y 2 z 2 (x y z )2 2(xy yz zx)
eday (x y z )2 0 RKb;cMnYnBit x , y , z enaHeK)an
x 2 y 2 z 2 2( xy yz zx ) (*)
yk x b a c , y c b a , z a c b enaHeK)an
ab
bc
ac
xy yz zx

(b c)(c a ) (a b )(c a ) (a b )(b c)


ab(a b ) bc(b c) ac(c a )

(a b )(b c)(c a )

b(c a )(b c a ) ac(c a )


(a b )(b c)(c a )
(c a )(b c)(b a )

1
(a b )(b c)(c a )

tamTMnak;TMng (*) eKTaj)an x2 y 2 z 2 2(1) 2


a2
b2
c2
dUcenH

2 .
2
2
2
(b c )

(c a )

eroberogeday lwm pln

(a b )

- TMBr 97 -

KNitviTaCMuvijBiPBelak
lMhat;TI40
cUrRsaybBaak;vismPaB
a 2 (1 b )2 b 2 (1 c)2 c2 (1 a )2

cMeBaHRKb;cMnYnBit a , b , c .
dMeNaHRsay
tamvismPaB Minkowsky eKman

3 2
2

x12 y12 x 22 y 2 2 x 3 2 y 32 ( x1 x 2 x 3 )2 ( y1 y 2 y 3 )2

yk x1 a , x2 b , x3 c
nig y1 1 b , y 2 1 c , y 3 1 a nig s a b c
eK)an (x1 x2 x3 )2 (y1 y 2 y 3 )2 s2 (3 s)2 92
eRBaH s2 (3 s)2 2s2 6s 9 12 [(2s 3)2 9] 92
dUcenH
a 2 (1 b )2 b 2 (1 c)2 c2 (1 a )2

eroberogeday lwm pln

3 2
2

- TMBr 98 -

KNitviTaCMuvijBiPBelak
lMhat;TI41
eK[RtIekaN ABC mYyEkgRtg; C . D nig E CacMNucBIreRCIserIs
enAelIGIub:UetnUs Edl BC BD nig AC AE .
F nig G CacMeNalEkgn D nig E elIRCug AC nig BC erogKa .
cUrRsaybBaak;fa DE DF EG .
dMeNaHRsay
RsaybBaak;fa DE DF EG

eyIgsg;km<s; CN rYcPab; CD nig CE .


eKman BC BD enaH BCD CaRtIekaNsm)atkMBUl B
eK)an BCD BDC Et BCD BCN NCD
nig BDC DCA A
eroberogeday lwm pln

- TMBr 99 -

KNitviTaCMuvijBiPBelak
ehtuenH BCN NCD DCA A (1)
eday CN AB enaH BCN A ( 2) mMubMeBjnmMu NCA
tam (1) nig (2) eKTaj)an NCD DCA
ehIy CND CFD 90o enaHeKTajRtIekaN CND nig CFD
CaRtIekaNEkgb:unKa . eKTaj)an DN DF .
RsaybMPWtamrebobdUcKaeK)an EN EG .
dUcenH DE DN NE DF EG .

eroberogeday lwm pln

- TMBr 100 -

KNitviTaCMuvijBiPBelak
lMhat;TI42
eKyk D CacMnucmYynRCug BC rbs;RtIekaN ABC ehIy E nig F
CacMeNalEkgn B nig C elI AD .
ebI R CacMnuckNaln BC enaHRsayfa RE RF
dMeNaHRsay
Rsayfa RE RF

bnay CF FG nig BE EH rYcPab; AG , BG , AH nig CH .


eKman BE AE nig BE EH sMNg;enaH ABE AHE
eK)an AB AH nig BAE HAE .
eroberogeday lwm pln

- TMBr 101 -

KNitviTaCMuvijBiPBelak
RsaydUcKaEdreK)an ACF AGF
eK)an AC AG nig CAF GAF .
eFIVpldk CAF BAF GAF AHE
eKTaj)an BAG CAH enaH AGB ACH
Cavi)akeKTaj BG CH .
kgRtIekaN CBG eKman R nig F CacMnuckNaln CB nig CG
eK)an RF 12 BG )atmFm
dUcKaEdrkgRtIekaN CBH eK)an RE 12 CH .
eday BG CH dUcenH RE RF .

eroberogeday lwm pln

- TMBr 102 -

KNitviTaCMuvijBiPBelak
lMhat;TI43
eK[kaer ABCD mYy . E CaRbsBVrvagGgt;RTUgTaMgBIr ehIy N CacMnuc
mYysitelI AE .
cUrRsayfa AB2 BN 2 AN.NC nig AN2 NC2 2BN 2
dMeNaHRsay
Rsayfa AB2 BN 2 AN.NC

eKman AC BD Ggt;RTUgnkaer enaH AE BE


tamRTwsIbTBItaKrcMeBaH AEB nig NEB
AB 2 AE2 EB 2 ( AN NE)2 EB 2 (1)
BN 2 NE2 EB 2 ( 2)

eroberogeday lwm pln

- TMBr 103 -

KNitviTaCMuvijBiPBelak
dksmIkarBIrenHGgnigGgeK)an
AB 2 BN 2 AN 2 2AN.NE AN.( AN 2NE)

Et AN 2NE AE NE NE EC NC
dUcenH AB2 BN 2 AN.NC .
Rsayfa AN2 NC2 2BN 2
eKman AN2 ( AE NE)2 AE2 NE2 2AE.NE (3)
ehIy NC2 (NE EC)2 (NE AE)2 eRBaH EC AE
NC2 NE2 AE2 2 AE.NE (4)

bUksmIkar (3) nig (4) eK)an


AN 2 NC2 2NE2 2 AE2

Et AE BE

dUcenH AN2 NC2 2(NE2 BE2 ) 2BN 2 .

eroberogeday lwm pln

- TMBr 104 -

KNitviTaCMuvijBiPBelak
lMhat;TI44
eK[ ABC CaRtIekaNmYyehIy D CaeCIgnkm<s;KUsBIkMBUl A .
yk E nig F sitenAelIbnat;kat;tam D edaydwgfa AE Ekgnwg BE
ehIy AF Ekgnwg CF Edl E nig F xusBI D .
yk M nig N CacMNuckNalnGgt; BC nig EF erogKa .
cUrRsayfa AN Ekgnwg NM ?
dMeNaHRsay
Rsayfa AN Ekgnwg NM
A
F

Q
N

E
B

eroberogeday lwm pln

- TMBr 105 -

KNitviTaCMuvijBiPBelak
tag (w1 ) nig (w 2 ) CargVg;manGgt;piterogKa AB nig AC .
eK)an E (w1 ) eRBaH AE BE ehIy F (w 2 ) eRBaH AF CF
tag P nig Q CacMnucRbsBVrvagbnat; ( AN) CamYy (w1 ) nig (w 2 )
erogKa .
cMnuc N enAeRkArgVg; (w1 ) enaHeK)an NE.ND NQ.NA (1)
eRBaH N CacMNuckNaln EF .
cMeBaHrgVg; (w 2 ) eKman NF.ND NP.NA (2)
tam (1) & (2) eKTaj)an NQ.NA NP.NA b NQ NP
eday MB MC enaHeK)an MN // PC ehIyeday AP PC
dUcenH AN NM .

eroberogeday lwm pln

- TMBr 106 -

KNitviTaCMuvijBiPBelak
lMhat;TI45
rkGnuKmn_ f : IR IR Edlepgpat;
x, y IR : f ((x y)2 ) x 2 2yf (x) f (y)

dMeNaHRsay
rkGnuKmn_ f
yk y x eK)an f (0) x 2 2xf ( x ) f ( x )2 ( x f ( x ))2
ebI x 0 enaH f (0) f 2 (0) naM[ f (0) 0 b f (0) 1
-krNI f (0) 0 eK)an ( x f ( x ))2 0 enaH f ( x ) x
epgpat;
f (( x y )2 ) ( x y )2
x 2 2 yf ( x ) f ( y )2 x 2 2 yx y 2 ( x y )2

eK)an f (( x y)2 ) x 2 2yf ( x ) f ( y) 2 ( x y)2 Bit


dUcenH f ( x ) x RKb; x IR
-krNI f (0) 1 eK)an ( x f ( x ))2 1 enaH f ( x ) x 1
b f ( x ) x 1 .
epgpat;
-cMeBaH f (x) x 1

eroberogeday lwm pln

- TMBr 107 -

KNitviTaCMuvijBiPBelak
f ((x y)2 ) (x y)2 1
x 2 2yf (x) f (y) x 2 2y(x 1) (y 1)2
2

(x y)2 1

eK)an f (( x y)2 ) x 2 2yf ( x ) f ( y) 2 ( x y)2 1 Bit


dUcenH f ( x ) x 1 RKb; x IR
-cMeBaH f ( x ) x 1
f ((x y)2 ) (x y)2 1
x 2 2yf (x) f (y) x 2 2y(x 1) (y 1)2
2

(x y)2 1

eK)an f (( x y)2 ) x 2 2yf ( x ) f ( y) 2


dUcenH f ( x ) x 1 minEmnCacemIy .
srubmkeK)an f ( x ) x b f ( x ) x 1 RKb; x IR .

eroberogeday lwm pln

- TMBr 108 -

KNitviTaCMuvijBiPBelak
lMhat;TI46
cMeBaHRKb;cMnYnBitviCman a ,b ,c cUrRsaybBaak;fa
3

(a 2 b2 )(b2 c2 )(c2 a 2 ) 3
(a b)(b c)(c a )
abc
8
6

dMeNaHRsay
bgajfa

(a 2 b2 )(b2 c2 )(c2 a 2 ) 3
(a b)(b c)(c a )
abc
8
x 2 y2
CadMbUgeyIgRtUvRsayfaRKb; x , y 0 eK)an x y 2 xy
eKman x y 2 xy enaH ( x y)2 4xy b ( x y)2 4xy 0
3

elIkCakaer [( x y)2 4xy]2 0


( x y )4 8xy( x y )2 16 x 2 y 2 0
( x y )4 8xy[( x y )2 2 xy] 8xy( x 2 y 2 )
( x y )2
x 2 y2
x 2 y2
eKTaj ( x y) 4 xy 2 b 2 2 xy 2
2
x 2 y2
x 2 y2
2 ( x y)
eday 2 xy. 2 ( 2 xy ) 2
2
( x y )2
x 2 y2
2 ( x y)
eK)an 2 ( 2 xy) 2
2

eroberogeday lwm pln

- TMBr 109 -

KNitviTaCMuvijBiPBelak
x 2 y2
naM[ x y 2 xy .
a b p ab
tamvismPaBxagelI eKTaj b c q bc

a c r ac
a 2 b2
b2 c2
a 2 c2
Edl p 2 , q 2 nig r 2
eK)an (a b)(b c)(c a ) ( p ab )( q bc )( r ac )

b (a b)(b c)(c a ) pqr m n abc


Edl m acpq bcpr abqr 33 a 2b2c2p2q 2r 2
nig n abc2p a 2bcq ab2cr 33 a 4b4c4pqr
eday pqr m n abc (6 pqr 3 abc )3
eKTaj (a b)(b c)(c a ) (6 pqr 3 abc )3
b 3 (a b)(b c)(c a ) 6 pqr 3 abc
(a 2 b2 )(b2 c2 )(c2 a 2 ) 3
3
6
dUcenH (a b)(b c)(c a)
abc
8

eroberogeday lwm pln

- TMBr 110 -

KNitviTaCMuvijBiPBelak
lMhat;TI47 Greece National Olympiad 2011
eK[ a , b , c CacMnYnBitviCmanEdlmanplbUkesI 6 .
cUrkMNt;tmGtibrman S a 2bc b 2ca c 2ab
dMeNaHRsay
kMNt;tmGtibrman S 3 a 2 2bc 3 b2 2ca 3 c2 2ab
tag u 3 a 2 2bc , v 3 b2 2ca nig w 3 c2 2ab
eKman u3 v3 w 3 a 2 2bc b2 2ca c2 2ab (a b c)2
Ettamsmtikm a b c 6 enaH u3 v3 w 3 36 (1)
ehIy S u v w (2)
cMeBaHRKb; x 0 eyIgeRCIserIs f ( x ) x3
eKman f ' ( x ) 3x 2 nig f '' ( x ) 6x 0 enaHtamvismPaB Jensen eK)an
3

f ( u ) f ( v ) f ( w ) 3f (

eK)an

uvw
) , u , v, w 0
3
3

1
uvw
3
u v w 3
( u v w ) (3)
3
9

S3
36
9

naM[ S 3 3 12
tamTMnak;TMng (1), (2) & (3) eK)an
dUcenHtmGtibrman S 3 a 2 2bc 3 b2 2ca 3 c2 2ab
esInwg Smax 3 3 12 EdlRtUvnwg a b c 2 .

eroberogeday lwm pln

- TMBr 111 -

KNitviTaCMuvijBiPBelak
lMhat;TI48 USAMO 1989
cMeBaHRKb;cMnYnKt;viCman n eK[
1 1
1
....
n
2 3
Tn S1 S 2 S 3 .... Sn

Sn 1

Un

T1 T2 T3
T

.... n
n1
2
3
4

cUrkMNt;edayeFIVdMeNaHRsay nUvcMnYnKt; 0 a, b, c, d 1000 000


edaydwgfa T1988 aS1989 b nig U1988 cS1989 d .
dMeNaHRsay
kMNt;cMnYnKt; a , b , c , d
eyIgnwgRsaytamkMeNInfaRKb; n 2 : Tn 1 nSn n
-cMeBaH n 2 : T1 2S2 2 2(1 12 ) 2 1 S1 Bit
-]bmafa Tn 1 nSn n Bit
eyIgnwgRsayfa Tn (n 1)Sn 1 (n 1) Bit
eKman Tn S1 S2 ... Sn 1 Sn Tn 1 Sn
Tn nS n n Sn (n 1)Sn n
eday Sn 1 Sn n 1 1 enaH Sn Sn 1 n 1 1

eroberogeday lwm pln

- TMBr 112 -

KNitviTaCMuvijBiPBelak
eK)an Tn (n 1)(Sn 1 n 1 1) n (n 1)Sn 1 (n 1) Bit
dUcenH n 2 : Tn 1 nSn n
yk n 1989 eK)an T1988 1989 S1989 1989
eday T1988 aS1989 b enaHeKTaj a 1989 , b 1989 .
n
T1 T2 T3
Tn
ehIy Un 2 3 4 ... n 1 ( kTk 1)
k 1

Un

(k 1)Sk 1 (k 1)

k 1
k 1
n

(Sk 1 1)

k 1

S 2 S 3 .... Sn 1 n
Tn 1 S1 n
(n 1)Sn 1 (n 1) 1 n
(n 1)Sn 1 2(n 1)

yk n 1988 eK)an U1988 1989S1989 3978


eday U1988 cS1989 d enaH c 1989 , d 3978
dUcenH a 1989 , b 1989 , c 1989 , d 3978 .

eroberogeday lwm pln

- TMBr 113 -

KNitviTaCMuvijBiPBelak
lMhat;TI49 (Japan Mathematical Olympiad Finals 2010)
eK[ x , y , z CacMnYnBitviCman . cUrbgajfa
1 yz zx 1 zx xy 1 xy yz

1 .
(1 x y) (1 y z) (1 z x)
2

dMeNaHRsay
tamvismPaB Cauchy Schwarz
(a1b1 a 2b 2 a 3b 3 )2 (a12 a 2 2 a 3 2 )(b12 b 2 2 b 3 2 )

x
y
,a 3 yz ,b 3
z
z
eK)an (1 x y)2 (1 xz yz)(1 xz yz )
eKTaj)an (11 xzx y)yz2 x yz z (1)
RsaydUcKaEdr (11 zxy z)xy2 x xy z (2)
1 xy yz
y

(3)
nig (1
z x)2 x y z

yk a

b1 1 ,a 2 xz ,b 2
1

eFIVplbUkvismPaB (1), (2) & (3) eK)an

1 yz zx 1 zx xy 1 xy yz x y z

1
(1 x y)2 (1 y z)2 (1 z x)2 x y z
1 xy yz
1 .
dUcenH (11 yzx yzx)2 (11 zxy zxy)2 (1
2
z x)

eroberogeday lwm pln

- TMBr 114 -

KNitviTaCMuvijBiPBelak
lMhat;TI50 China Team Selection Test 2002
eK[sIVt a 1, a 5 nig a a a
1

n 1

n 1

an a
2

2
n 1

n 2

cUrkMNt;tYTUeTAnsIVt {a } CaGnuKmn_n n .
dMeNaHRsay
kMNt;tYTUeTAnsIVt {a } CaGnuKmn_n n
eKman a a a
n 2
n

n 1

eK)an
1
an2 1
1
an2 1
1

n 1

an 2 an2 1 1

an 2 an2 1 1

an 2an2 1

1
1
1

an 2 an2 1 an 2an2 1

1
an2 1

1
1
1
)
(1
)

an 2 an2 1
an 2

1
1
)(1
)

an2 1
an2
an2 1
b 1 a12 (1 a12 )(1 a12 ) (*)
n 2
n
n 1
tag bn ln(1 a12 ) enaH bn1 ln(1 a12 )
n
n 1
1

(1
(1

eroberogeday lwm pln

- TMBr 115 -

KNitviTaCMuvijBiPBelak
ehIy

bn 2 ln(1

1
an2 2

) (**)

yk (*) CMnYskg (**) eKTTYl)an b b b


smIkarsmal; q q 1 0 manbs q 1 2 5
n 2

n 1

, q2

1 5
2

1 5 n
1 5 n
)

(
) Edl , kMNt;dUcxageRkam
n
2
2
cMeBaH n 1 b1 . 1 2 5 . 1 2 5
cMeBaH n 2 : b2 3 2 5 3 2 5
26
eday b1 ln(1 a12 ) ln 2 nig b2 ln(1 a12 ) ln 25
1
2
1 5
1 5

ln 2

2
eK)an 2
3 5 3 5 ln 26
2
2
25

eKTaj 12 ln 2 513 1 ln 100

3
5

1 2 13 1 100
nig 2 ln 5 ln 3
5

ehIy bn ln(1 a12 ) eKTaj an b1n .


n
e 1

eK)an b

eroberogeday lwm pln

- TMBr 116 -

KNitviTaCMuvijBiPBelak
lMhat;TI51 China Team Selection Test 2006
eK[ x , x ,...., x CacMnYnBitviCmanEdl x 1 . cUrRsayfa
n

i 1

xi

n
1
i1
1 xi

n2

n1

dMeNaHRsay
tag 1 x y eKTaj x
i

vismPaB
n

xi

i 1

i 1

n
y i 1
i 1

i 1

y i 1 nig

n1

yi

i 1

n
1
n2
i1

1 xi
n1

1
n2

yi
n1

tamvismPaB Cauchy Schwarz a b


eKyk a 1 ,a y 1 i 2
1

ehIy

i1

smmUl

i 1

Edl y

(a i

i 1

). (b i 2 )
i 1

b1 y 1 1 , b i

1
, i 2
n

eK)an

n 1
1
1 n

. y i 1 (y i 1) y 1 1
i2 n
n

n i2

eday (y 1) n 1 y (n 1) 2 y
ehIy n1 n n 1 enaHeK)an
n

i2

i2

eroberogeday lwm pln

- TMBr 117 -

KNitviTaCMuvijBiPBelak
n 1
1 2n
1
1 n

(y
1)
y
1
(
y
)(y
)

i
i
i

i
2
i
2
n
n
n
n

y 12

2(n 1)
2n 1
y1
n
n2

eKTaj in1 1 . y i 1 y12 2(nn 1) y1 2nn2 1


n

b in1 y i 1 n y12 2(nn 1) y1 2nn2 1


b 1 in1 y i 1 n y1 2nn 2 2nn2 1 . y1
y1
1
2

dUcKaEdreKTaj)an
1
y2

i 1

yi 1 n y2

2n 2 2n 1 1
.

2
y2
n
n

1
yn

i 1

yi 1 n yn

2n 2 2n 1 1
.

n
n2 yn

edayeFIvviFIbUkGg nig GgnbNavismPaBxagelIeK)an


n

i 1

yi 1 .

i 1

n
1
2n 2 2n 1 1
n yi

. (*)
2
i
1

n
n
yi
yi

tamvismPaB Cauchy Schwarz eKman


n

i 1

yi

2n 2 2n 1 1
. n

2
n2 y i

eroberogeday lwm pln

(y i

i 1

2n 2 2n 1 1

. )
n
n2 y i

- TMBr 118 -

KNitviTaCMuvijBiPBelak
eday ( y
n

i 1

eK)an
n

i 1

ehIy

2n 2
) (n 1) 2n 2 n 1
n

yi

2n 2 2n 1 1
2n 1 n 1

n
n

.
2
n2 y i
n 2 i 1 y i

1 (1 1 ... 1)2
n2

n
i 1 y
n1
yi
i
n

i 1

eKTaj
n

i 1

2n 2 2n 1 1
2n 1 n 2
yi

. n n 1
.
2
n2 y i
n2 n 1

yi

i 1

2n 2 2n 1 1
n

n
.
(**)
2
n2 y i
n1

n 1
n2
tam (*) nig (**) eKTaj i1 y i 1 i1
yi
n1

n
n
1
n2
dUcenH i1 xi i1
.

1 xi
n1

Bit

eroberogeday lwm pln

- TMBr 119 -

KNitviTaCMuvijBiPBelak
lMhat;TI52 China Team Selection Test 2005
eK[ a ,b ,c CabIcMnYnBitminGviCman Edl ab bc ca 13 .
1
1
1

3 .
cUrbgajfa a bc
1 b ca 1 c ab 1
dMeNaHRsay
a
b
c
tag S a bc

1 b ca 1 c ab 1
2

1
1
1

a 2 bc 1 b 2 ca 1 c 2 ab 1
a2
b2
c2
eKman S a3 abc a b 3 cab b c3 abc c
tamvismPaB Cauchy Schwarz eK)an

nig

(a b c)2
S 3
a b 3 c 3 3abc a b c

Et a b c 3abc (a b c)(a b c ab bc ca)


bc
1
eK)an S a b ca ab
eday
ab bc ca
bc ca 1
3
3

abc
1
.

2
b 2 c 2 2ab 2bc 2cc a b c
ca ab bc ca ab bc ca
ma:geTot T3 aba2 bc
2
2
bc 1
b ca 1
c ab 1
ca a(a b c)
1
ehIy aba2 bc
2
2
1
bc 1
a bc 1 a bc 1

enaH S a

eroberogeday lwm pln

- TMBr 120 -

KNitviTaCMuvijBiPBelak
eK)an

T
ab bc ca
2
3 cyc a bc 1
a(a b c)
1
[ 2
2
1]
cyc a bc 1
a bc 1
(a b c)S T 3 (a b c).

1
T3
abc

eRBaH S a b1 c sRmayxagelI
eKTaj T T3 2 b T 3 .
dUcenH a

1
1
1

3
2
2
2
bc 1 b ca 1 c ab 1

eroberogeday lwm pln

- TMBr 121 -

KNitviTaCMuvijBiPBelak

eroberogeday lwm pln

- TMBr 122 -

KNitviTaCMuvijBiPBelak
CMBUkTI3

lMhat;Gnuvtn_
!> eK[sIVtncMnYnBit (x ) kMNt;eday
n

cMeBaHRKb; n 1 cUrRsaybBaak;fa
1

1
2n 1

x1 2

xn 1
x
x
n 1
n

1
1 1
1
.... 1 n
x1 x 2
xn
22

2
@> eK[ a , b ,c CacMnYnBitviCmanEdl abc 1
1
1
1

1 .
ab1 bc1 ca1

.
. cUrbgajfa

#> cUrkMNt;tmGb,brmanplbUk

1
1
1
Sn log x1 (x 2 ) log x2 (x 3 ) ... log xn (x1 )
4
4
4
Edl x1 , x2 , ...., xn CacMnYnBitncenaH ( 14 ,1) .
$> cUrkMNt;GnuKmn_ f : IR IR ebIeKdwgfa
f (xf (x) f (y)) (f (x))2 y

eroberogeday lwm pln

cMeBaHRKb; x, y IR .

- TMBr 123 -

KNitviTaCMuvijBiPBelak
3x y

3
2
2

%> edaHRsayRbBnsmIkar xx 3yy


y
0

x2 y 2
n
1
^> KNnaplbUk k0 (n k)!(n
k)!

&> edaHRsaysmIkar 2(2 1)x (2 2)x 2


*> cUrbgajfa 12 . 43 .... 2n2n 1 1
3n
cMeBaHRKb;cMnYnKt;viCman n .
x

x2

x 0 a , x1 b
(> eK[sIVt x 1 (x 1 )
n 1 2 n 1 x

n
cUrbgajfa a.b 1 .

n 1

CasIVtxYb Periodic .

!0> cUrkMNt;RKb;cemIyBitrbs;smIkar 2 3
!!> eK[ {a } CasIVtmYyEdl a 2 , a
n

x 1

6x x 2 .
an 1

( n IN )
n 1
2 an
x

cUrkMNt;rUbmnGiucBIsIutmYyn (a ) .
!@> eK[ a ,a , ....,a CasIVtncMnYnBitEdl a
Edl n 1 . cUrbgajfa a (2 , 1) .
n

n 1

an 2 an 1

eroberogeday lwm pln

- TMBr 124 -

KNitviTaCMuvijBiPBelak
!#> cUrRsayfaRtIekaN ABC CaRtIekaNEkgluHRtaEt
sin A sin B sin C
2 .
cos A cos B cos C
!$> cUrkMNt;RKb;KUncMnYnKt;viCman (a,b,c) ebIeKdwgfa
a 1 , b 1 CacMnYnbzm nig (a 1)(b 1) c 1 .
!%> eK[ n 4 CacMnYnKt;viCman nigyk a ,a , ....,a CacMnYnBitEdl
a a ... a 1 . cUrbgajfa
2

a1
a2
an
4
2

...

(a
a

...

a
a
)
1
1
n
n
a 22 1 a 23 1
a12 1 5

!^> eK[ a , b ,c CacMnYnBitviCmanmanplbUkesI # .


cUrRsayfa a b c ab bc ca .
!&> cUrRsaybBaak;fa

2|sin x cos x | 3 | sin x cos x | RKb; x (0, )


2
Edl n m CacMnYnKt;viCman .
!*> cUrkMNt;RKb;cMnYnKt;viCman a nig b edaydwgfa ab ba nig ba ba
CacMnYnKt;RBmKa .
!(> eK[ x , y , z CacMnYnBitviCmanEdl 1x y1 1z 1 . cUrRsayfa
x yz y zx z xy xyz x y z .
n

eroberogeday lwm pln

- TMBr 125 -

KNitviTaCMuvijBiPBelak
@0> eK[ a , b , c CabIcMnYnBitviCmanEdl abc 1 .
abc

a2 b 2 c2
3

cUrbgajfa
@!> eK[ a , b , c CabIcMnYnBitxusKa .
a2
b2
cUrRsayfa

2
2
(b c )

(c a )

c2
2
2
(a b )

@@> eK[ a , b , c CabIcMnYnBitminGviCmanEdl a2 b2 c2 a b c


cUrRsayfa a2b2 b2c2 c2a2 ab bc ca .
@#> eK[ a , b , c CabIcMnYnBitminGviCman . cUrRsayfa
(a 2 ab b 2 )(b 2 bc c2 )(c2 ca a 2 ) (ab bc ca)3

@$> eK[ a , b , c CabIcMnYnBitminGviCmanEdl abc 1 . cUrRsayfa


c a3 b 3 a b 3 c3 b c3 a3
3

.
2
2
2
2
2
2
2
a b
b c
c a
@%> eK[ a , b , c CabIcMnYnBitviCman . cUrRsaybBaak;vismPaB
a3
b3
c3

3
1
3
3
3
3
3
a (b c )
b (c a )
c (a b )

@^> cMeBaHRKb;cMnYnBitviCman a , b , c cUrRsayfa


a
b
c
3

ab
bc
ca
2

eroberogeday lwm pln

- TMBr 126 -

KNitviTaCMuvijBiPBelak
@&> cMeBaHRKb;cMnYnBitminGviCman x , y , z cUrRsayfa
2(1 x 2 )(1 y 2 )(1 z 2 ) (1 x )(1 y )(1 z )(1 xyz )

@*> cMeBaHRKb;cMnYnBitminGviCman a, b, c, d cUrRsayfa


4(1 a a 2 )(1 b b 2 )(1 c c 2 )(1 d d 2 ) (1 abcd )2

@(> ebIsmIkar x4 ax3 2x2 bx 1 0 manbsy:agticmYy


CacMnYnBitenaHbgajfa a4 b4 8 .
#0> cUrRsaybBaak;fa
a
b
c
9

(b c)2 (c a )2 (a b )2 4(a b c)

cMeBaHRKb; a , b , c CacMnYnBitviCman .
a3 b 3 c3 a2 b 2 c2
#!> cUrRsaybBaak;fa 2 2 2 b c a
b
c
a
cMeBaHRKb; a , b , c CacMnYnBitviCman .
#@> RKb;cMnYnBitviCman a , b , c cUrRsaybBaak;fa

bc ca ab
b
c
a

a
b
c
b c c a a b
##> eK[ a , b , c CabIcMnYnBitviCmanEdl abc 1 .

cUrRsayfa (a b)(b c)(c a) 4(a b c 1)


#$> cUrbgajfa (a2 2)(b2 2)(c2 2) 9(ab bc ca)
cMeBaHRKb;cMnYnBitviCman a , b , c .
eroberogeday lwm pln

- TMBr 127 -

KNitviTaCMuvijBiPBelak
#%> eKyk a , b , c CabIcMnYnBitviCman . cUrbgajfa
(b c a ) 2
(c a b )2
(a b c )2 3

2
2
2
2
2
2
5
(b c ) a
(c a ) b
(a b ) c

#^> eK[ x , y , z , t CabYncMnYnBitviCman . cUrRsayfa


2

x y z t



1 .
x y y z z t t x
#&> eK[ a , b , c nig x , y , z CacMnYnBit . cUrRsaybBaak;fa
4(a 2 x 2 )(b 2 y 2 )(c 2 z 2 ) 3(bcx cay abz )2

#*> eK[bIcMnYnBit a 1 , b 1 , c 1 . cUrRsaybBaak;fa


1 a2
1 b2
1 c2
2 .

2
2
2
1 b c 1 c a 1 a b
#(> eK[bIcMnYnBitviCman a , b , c Edl abc 1 . cUrRsaybBaak;fa
1
1
1
3

.
3
3
3
8
(1 a )

(1 b )

(1 c)

$0> eK[bIcMnYnBitviCman a , b , c Edl


a 2 b 2 c 2 (a b c )2 4 .
cUrRsaybBaak;fa ab 12 bc 12 ca 12 3
(a b )
(b c )
(c a )

$!> eK[bIcMnYnBitviCman a , b nig c . cUrRsayfa


(a5 a 2 3)(b 5 b 2 3)(c5 c 2 3) (a b c)3

eroberogeday lwm pln

- TMBr 128 -

KNitviTaCMuvijBiPBelak
$@> cUrRsayfacMeBaHRKb;cMnYnBitviCman a , b , c eKmanvismPaB
1
1
1
1

.
3
3
3
3
3
3
a b abc b c abc c a abc abc
mm 1 nn 1
$#> eKyk a m n Edl m nig n CacMnYnKt;viCman .
m n
cUrRsayfa am an mm nn .

$$> eK[ x ; y ; z CabIcMnYnBitviCmanEdl xyz 1 .


cUrRsaybBaak;fa
x9 y 9
x x y y
6

3 3

y9 z9
y y z z
6

3 3

z 9 x9
z z x x
6

3 3

$%> eK[ a , b , c CacMnYnviCmanEdl ab bc ca 3 .


1
cUrbgajfa 1 a 1(b c) 1 b 1(c a) 1 c 1(a b) abc
2

$^> eK[ a nig b CaBIrcMnYnBitminGviCmanEdl a

cUrRsayfa

b2 4

ab
2 1
ab2
x3 y 3 z 3
3
xyz | ( x y )( y z )( z x ) |
3
4
x;y;z0

$&> cUrRsayfa
cMeBaHRKb;
.
$*> eK[ a , b , c CaRbEvgRCugnRtIekaNmYy .
cUrRsayfa b aa c a bb c a bc c 3 .
eroberogeday lwm pln

- TMBr 129 -

KNitviTaCMuvijBiPBelak
$(> eK[ a , b , c CabIcMnYnBitEdlepgpat; a b c 3 .
cUrbgajfa | a | | b | | c | abc 4 .
%0> eK[ a , b , c CabIcMnYnBitviCmanEdl abc 1 .
cUrbgajfa 1 a b3 c ab bc6 ca .
%!> eK[ a 1 nig b 1 .
cUrbgajfa log a log b 2 log ( a b ) .
2
%@> eK[ x , y , z CabIcMnYnBitviCmanEdlepgpat; xyz 1 .
cUrbgajvismPaB
2

( x y 1) 2 ( y z 1) 2 ( z x 1) 2

xyz
z
x
y

%#>eK[ a , b , c CacMnYnBitviCman .
a 2b(b c) b 2c(c a) c 2a(a b )
cUrbgajfa a b b c c a 0 .
%$> eK[ a , b , c CabIcMnYnBitviCman . cUrbgajfa
a5 b 5 c5 (a b c)5 10
2

(
a

c
)
a 3 b 3 c 3 (a b c ) 3 9
%%> cMeBaHRKb;cMnYnBit x (0 , 4 )
cUrbgajfa (cos x)cos x (sin x)sin x .

eroberogeday lwm pln

- TMBr 130 -

KNitviTaCMuvijBiPBelak
%^> eK[ a , b , c , d CabIcMnYnBitEdlepgpat;vismPaB
(a 2 b 2 1)(c 2 d 2 1) (ac bd 1)2

cUrbgajfa a b 1 nig c d 1 .
%&> eK[ a , b , c CabIcMnYnBitviCmanEdl ab bc ca 1 .
cUrkMnt;tmGb,brmankenSam E a a b bb c c c a .
%*> eK[ x1 , x2 , ..., xn Edl n 2 CacMnYnBitviCmanEdlepgpat;
2

1
1
1
1

....

x1 1998 x 2 1998
xn 1998 1998

cUrbgajfa x1n.x2 1...xn 1998 .


%(> eK[ x , y , z CabIcMnYnBitviCmanEdl xyz x y z .
cUrRsayfa 1xzy 1yxz 1zyx 227
.
xyz
^0> eKyk a , b , c CacMnYnBitviCmanEdl ab bc ca abc
b 4 c4
c4 a4
a4 b4
cUrbgajfa ab(a3 b3 ) bc(b3 c3 ) ca(c3 a3 ) 1 .
^!> eKyk x , y , z CacMnYnBitviCmanEdl xyz 1
cUrbgajfa yz1 z zx1 x xy1 y 32 .
n

eroberogeday lwm pln

- TMBr 131 -

KNitviTaCMuvijBiPBelak
^@> cMnYnBit a, , b , c , x , y , z epgpat; a b c 0
nig x y z 0 . cUrbgajfa
a2x2
b2y 2
c2z 2
3

(by cz )(bz cy ) (cz ax )(cx az ) (ax by )(ay bx ) 4


3 3 xyz
x
y
z

1 x 1 y 1 z 1 3 xyz

^#> cUrbgajfa
cMeBaHRKb; 0 x , y , z 1 .
^$> cUrbgajfacMeBaHRKb; a , b , c CacMnYnBitviCmaneK)an
a
b
c

1 .

2
2
2
a 8bc

b 8ca

c 8ab

bgajfa 2 a 2 b 2 c 1
a 8bc
b 8ca
c 8ab
^%> cUrbgajfacMeBaHRKb; a , b , c CacMnYnBitviCmaneK)an
(a 1)(b 1)2
3 c a 1
3

2 2

(b 1)(c 1)2
3 a b 1
3

2 2

(c 1)(a 1)2

3 b c 1
3

^^> eK[ x , y , z CacMnYnBitviCmanEdl


cUrRsayfa
x 2 yz
2x ( y z )
2

y 2 zx
2y ( z x )

eroberogeday lwm pln

2 2

abc3

x y z 1

z 2 xy
2z ( x y )
2

- TMBr 132 -

KNitviTaCMuvijBiPBelak
^&> ebIcMnYnBitviCman a , b , c epgpat; a2 b2 c2 3
enaHcUrbgajfa
a2
b2
c2
(a b c ) 2

2
2
2
12
2bc
2ca
2ab

.
^*> eKmancMnYnBitviCman a , b , c epgpat; a b c 1 .
b ca c ab 3
cUrbgajfa aa bc
.

bc b ca c ab 2
^(> eK[ a , b , c Ca cMnYnBitviCman.
cUrbgajfa a 1 bc b 1 ca c 1 ab 12 ( ab1 bc1 ca1 ) .
&0> eK[ a , b , c , d CacMnYnBitviCman .
2

a4
b4
c4
d4
S

2
2
2
2
2
2
(a b)(a b ) (b c)(b c ) (c d)(c d ) (d a)(d 2 a 2 )

cUrbgajfa S a b 4 c d .
&!> eK[cMnYnBitviCman x1 , x2 , ...., xn ehIyeKtag
Sn x1 x 2 ... xn . cUrRsayfa
S2
Sn
(1 x1 )(1 x 2 )....(1 xn ) 1 S ...
2!
n!
&@> cMeBaHRKb;cMnYnKt;viCman n nig cMeBaHRKb;cMnYnBitviCman

epgpat; a1a 2a 3 ....an 1 .


n
ai
1 n 1
cUrbgajfa 4 2 a .

a1 , a 2 , ...., a n

i 1

eroberogeday lwm pln

ai 3

i 1 i

- TMBr 133 -

KNitviTaCMuvijBiPBelak
&#> eK[ a , b , c CacMnYnBitviCman edaydwgfa
1 1 1
a b c . cUrbgajfa
a b c
1
( 2a b c)

1
(a 2b c)

1
(a b 2c)

3
16

&$> smIkar x3 ax2 bx c 0 manbbICacMnYnBitviCman

mincaM)ac;xusKa .
cUrkMNt;tmGb,brmaEdlGacn 13a2abbc bc .
&%> cUrRsayfa
2
2 2
2

(
a
b
)(
a
ab
b
) 2 2 2 2 1
1
1
4
(
a
b
c
)(
)

2
3
ab bc ca
Cyc

cMeBaHRKb;cMnYnBitviCman a , b , c .
&^> ebI a , b , c CargVas;RCugnRtIekaNmYy ehIy r CakaMrgVg;
carwkkgnRtIekaNenaHcUrRsayfa a12 b12 c12 41r 2 .
&&> eK[ a , b , c CacMnYnBitviCmanehIyepgpat;lkxN
1 1 1
16(a b c) . cUrRsaybBaak;fa
a b c
1
(a b 2a 2c)3

eroberogeday lwm pln

8
1

(b c 2a 2b)3 (c a 2b 2c)3 9
1

- TMBr 134 -

KNitviTaCMuvijBiPBelak
&*> eK[ a , b , c CacMnYnBitviCmanEdl a b c 1 .

cUrbgajfa ab 2 bc 2 ca 2 83 .
1 a
1b
1 c
&(> eK[ a , b , c CabIcMnYnBitminGviCman nig x , y , z
CabIcMnYnBitviCmanedaydwgfa a b c x y z .
a3 b 3 c3
cUrRsaybBaak;fa 2 2 2 a b c .
x

*0> eK[ a , b , c 0 . cUrRsaybBaak;fa


a 3 4(b 3 c 3 ) b 3 4(c 3 a 3 ) c 3 4( a 3 b 3 ) 9

ca
2
bc
ab
*!> eKeGaybIcMnYnBitviCman a , b , c . cUrRsayfa
a3
b3
c3
3

(b c )3 (c a )3 (a b )3 8

*@> cMeBaHRKb;cMnYnBitviCman a , b , c eKkMNt;tag A a b3 c


3
.
1 1

a b c
3
A
cUrRsayfa G 14 43 . HA ?

G 3 abc

nig H 1

eroberogeday lwm pln

- TMBr 135 -

KNitviTaCMuvijBiPBelak
*#> eK[cMnYnBit x1 , x2 , y1 , y 2 , z1 , z 2 epgpat; x1 0 , x2 0
x1y1 z12 0 nig x 2y 2 z 2 2 0 .
cUrRsaybBaak;fa
8
( x1 x 2 )( y1 y 2 ) ( z1 z 2 )

1
x1y1 z1

1
x2y 2 z 2

(*)

*$> eK[RtIekaN ABC mYymanmMukgCamMuRsYc . cUrRsayfa


3 2
.
cos A cos B cos C
2
*%> eK[ x ; y ; z CacMnYnBitviCmanEdl xyz 1 . cUrRsayfa

1
1
1
1

( x 1)2 y 2 1 ( y 1)2 z 2 1 ( z 1)2 x 2 1 2

*^> eK[ a ; b ; c CacMnYnBitviCmanEdl abc 1 .


cUrRsayfa
.
*&> eK[ a1 , a2 , ..., an 0 nig x1 , x2 , ..., xn 0 .
cUrRsaytamkMeNInfa
a
a
a
(a a ... a )
.

...

x
x
x
x x .... x
a(b 2 b ) b(c 2 c ) c(a 2 a ) 0

**> eK[ a ; b ; c CabIcMnYnBitviCmanEdl abc 1 .


eroberogeday lwm pln

- TMBr 136 -

KNitviTaCMuvijBiPBelak
cUrbgajfa
1
1
1
3

a 3 (b c ) b 3 (c a ) c 3 (a b ) 2

*(> eK[ x ; y ; z 0 . cUrRsaybBaak;fa


x
y
z
1

x 2y 3z y 2z 3x z 2x 3y 2

(0> eK[sIVt a1 ; a2 ; ....; an epgpat;lkxN


a1 0 ; | a 2 || a1 1 |; ... nig | an || an 1 1 | .
cUrbgajfa a a n ... a 12 .
(!>eK[ a ; b ; c CabIcMnYnBitviCman .
cUrbgajfa
a
b
c
abc

) .
1 1 1 2 (1 3
b
c
a
abc

(@>eK[ a ; b ; c CaRbEvgRCugrbs;RtIekaNmYy .
cUrbgajfa

abc bca cab a b c

(#>eK[RtIekaN ABC mYymanmMukgCamMuRsYc . cUrbgajfa


sin 2 A sin 2 B sin 2 C 2 3 sin A sin B sin C .
($>eK[ A ; B ; C CamMukgrbs;RtIekaN ABC mYy .
eroberogeday lwm pln

- TMBr 137 -

KNitviTaCMuvijBiPBelak
cUrbgajfa cot A2 cot B2 cot C2 3 3
(%>eK[ A ; B ; C CamMuRsYckgrbs;RtIekaN ABC mYy .
cUrbgajfa (1 tan A)(1 tan B)(1 tan C) (1 3 ) .
(^>eK[ a ; b ; c CabIcMnYnBitviCman . cUrRsayfa
(a 2)(b 2)(c 2) 9(ab bc ca) .
(&>RbsinebI xyx (1 x)(1 y )(1 z ) Edl 0 x ; y ; z 1
cUrbgajfa x(1 z ) y(1 x) z(1 y ) 43 .
(*>eK[ a ; b ; c CaRbEvgRCugrbs;RtIekaNmYyEdlman
brimaRtesI 2 .
cUrRsayfa 32 a b c 2abc 2 .
((>eK[ x ; y ; z CabIcMnYnBitviCmanEdl x y z 1 .
cUrRsaybBaak;fa 1x 1 y1 1 1z 1 8 .

!00> eK[ a ; b ; c CabIcMnYnBitviCmanEdl a b c 1


1
1
2(a 3 b 3 c 3 )
1
bgajfa a2 b2 c2 3
abc
3

!0!> eK[ a , b , c CabIcMnYnBitviCman . cUrRsaybBaak;fa


eroberogeday lwm pln

- TMBr 138 -

KNitviTaCMuvijBiPBelak
a2
2a2 ab b2

b2
2b2 bc c2

c2
2c2 ca a2

abc
2

!0@> eK[bIcMnYnBitviCman a , b , c . cUrRsaybBaak;fa

29a 3 b 3 29b 3 c 3 29c3 a 3

4(a b c)

2
2
2
ab 6a
bc 6b
ca 6c
!0#> eK[ a , b , c CabIcMnYnBitviCman . cUrRsaybBaak;vismPaB
b3
c3
abc
a3

3
a 2 ab b 2 b 2 bc c 2 c 2 ca a 2
!0$> eK[bIcMnYnBitviCman a , b , c Edl a b c 1 .

cUrRsaybBaak;fa
1
1
1
1

ab 2c2 2c bc 2a2 2a ca 2b2 2b ab bc ca


!0%> eK[RtIekaN ABC mYymanRCug BC a , CA b , AB c

ehIymMukg A , B , C CamMuRsYcbmMuEkg .
tag S CapRkLan ABC
cUrRsaybBaak;fa 14 14 14 9 2 .
a

!0^> cUrbgajfa

16 S
1
1
1
4

( x y )2 ( y z )2 ( z x )2 xy yz zx

cMeBaHRKb;cMnYnBitminGviCmanxusKa x , y , z .

eroberogeday lwm pln

- TMBr 139 -

KNitviTaCMuvijBiPBelak
!0&> cUrbgajfa

1
1 1

a b

1 1
1
1

c d
ac bd
cMeBaHRKb;cMnYnBitviCman a , b , c , d .

!0*> eK[ a , b , c CacMnYnBitviCman . cUrRsaybBaak;fa


( 2a b c)2
( 2b c a )2
( 2c a b )2
2
2
8
2
2
2
2
2a (b c)
2b (c a )
2c (a b )

!0(> eK[ a , b , x , y , z CacMnYnBitviCman .


cUrbgajfa ay x bz az y bx ax z by a 3 b
!!0> eK[ n 2 CacMnYnKt; ehIy x1 , x2 , ...., xn CacMnYnBitviCman
edaydwgfa x x .... x 1 . cUrkMNt;tmtUcbMputn
2

x1
x2
xn

...
x 2 x 3 ... xn x 3 ... xn x1
x1 x 2 ... xn1

!!!> eK[ n CacMnYnKt;viCman . cUrkMNt;tmGb,brmanplbUk


2
3
n
xn
x2
x3

.....
.
S x1
n
2
3
(Poland 1995 )

!!@> cUrbgajfacMeBaHRKb;cMnYnBitviCman a , b , c , d eKman


a4b b 4c c4d d 4a abcd(a b c d )

eroberogeday lwm pln

- TMBr 140 -

KNitviTaCMuvijBiPBelak
!!#> eK[ a , b , c CacMnYnBitviCman . cUrbgajfa
2
3

2
3

2
3

2a
2b
2c



3
b c c a a b
(MOP 2002 )

!!$> cUrbgajfacMeBaHRKb;cMnYnBitviCman a , b , c eKman


a2
b2
c2
3

(a b )(a c) (b c)(b a ) (c a )(c b ) 4

!!%> cUrbgajfacMeBaHRKb;cMnYnBitviCman a , b , c eKman


a 2b b 2c c 2a

3


a 2c b 2a c 2b
!!^> eK[ a , b , c , x , y , z CacMnYnBitviCmanEdl x y z 1
3

cUrbgajfa
ax by cz 2 (ab bc ca)( xy yz zx ) a b c

!!&> eK[ a , b , c CacMnYnBitviCmanEdl abc 1 .


cUrbgajfa 5 ba bc ac (1 a)(1 b)(1 c)
!!*> eK[ a , b , c CacMnYnBitviCman . cUrbgajfa
a3
b3
c3
1

2 2

2
2
2
2
2
2
2
2
2
2
(2a b )(2a c ) (2b c )(2b a ) (2c a )(2c b ) a b c

!!(> eK[ a , b , c CacMnYnBitviCman . cUrbgajfa


(a5 a 2 3)(b 5 b 2 3)(c5 c 2 3) (a b c)3

eroberogeday lwm pln

- TMBr 141 -

KNitviTaCMuvijBiPBelak
a2 b 2 c2 a b c

b 2 c2 a2 b c a

!@0> RKb; a , b , c 0 cUrRsaybBaak;fa


!@!> cUrRsayfaRKb; a , b , c 0 eKman

b 2 c2 a 2 c2 a2 b 2 a 2 b 2 c2 3

a( b c )
b(c a )
c(a b )
2

!@@> eK[ a , b , c 0 Edl a b c 1 . cUrRsayfa


ab c bc a ca b 1 ab bc ca
!@#> eK[ a , b , c 0 Edl abc 8 . cUrRsayfa
a2
(a 3 1)(b 3 1)

b2
(b 3 1)(c3 1)

c2
(c3 1)(a 3 1)

4
3

!@$>cUrRsayfaRKb;cMnYnBit 0 x1 , x2 , ....., xn 12 eKmanvismPaB


n

n 1

1 1
i 1 x i
x1 x 2 ... xn

!@%> bgajfaRKb; a , b , c 0 eK)an

a2
b2
c2
1

( 2a b )( 2a c) ( 2b c)( 2b a ) ( 2c a )( 2c b ) 3

!@^> bgajfaRKb; a , b , c 0 eK)an


3

4a 2
4b 2
4c2

4a 4b 4b 4c 4c 4a
ab bc ca
3

eroberogeday lwm pln

- TMBr 142 -

KNitviTaCMuvijBiPBelak
!@&> eKman a , b , c , x , y , z IR Edlepgpat;smPaB
(a b c)( x y z ) 3

nig (a b c )(x y z ) 4
cUrbgajfa ax by cz 0 .
!@*> eK[ a , b , c 1 Edl a21 1 b21 1 c2 1 1 1 .
2

cUrRsayfa a 1 1 b 1 1 c 1 1 1 .
!@(> bgajfaRKb; a , b , c 0 eKman
a

a2 b2
b 2 c2
c2 a2
!#0> eK[ n 2 CacMnYnKt;viCman .

3 2
2

cMeBaHRKb;cMnYnBitviCman a1 , a2 ,..., an Edl a1 .a2 ....an 1


cUrRsaybBaak;fa
a1 1
a 1
a 1
2
... n
a1 a 2 ... an
2
2
2
!#!> eK[ a , b , c , x , y , z IR Edl xy yz zx 3 .
cUrbgajfa a(bycz ) b(czax) c(axby ) 3 .
!#@> eK[ a , b , c 0 Edl a b c 3 .
cUrbgajfa aba 1 bcb 1 cac 1 32 .
2

eroberogeday lwm pln

- TMBr 143 -

KNitviTaCMuvijBiPBelak
!##> eK[ a1 , a2 , ...., an 0 . cUrRsaybBaak;fa
a1 a1a2 3 a1a2a3 ... n a1a2...an n a1 a2 a1 a2 ... an
...
a1.
n
2
n
!#$> cUrRsayfaRKb; a , b , c 0 eKman
ab bc ca bc ca ab ca ab bc 3

2
2
2
2
2
2
2
b c
c a
a b
!#%> eK[ a , b , c CacMnYnBitviCman . cUrbgajfa
a 2 bc b 2 ca c 2 ab b c a

b 2 ca c 2 ab a 2 bc a b c
!#^> bgajfaRKb;cMnYnBitviCman x , y , z Edl x y z 1

eKman

x ( y z )2 y ( z x )2 z ( x y )2 3

!#&> eK[ a , b , c CacMnYnBitminGviCman nig KanBIrcMnYnNaesIsUn.


ca bc ca ab ca ab bc 3

cUrRsayfa abc2 bc
2
a2
a2 b2
b 2 c2
!#*> cMeBaHRKb;cMnYnBitviCman a nig b cUrRsayfa
a b
2ab
1 2
2 2
2
b a
a b
!#(> eK[ a , b , c CargVas;RCugnRtIekaNmYy .
cUrRsayfa 3a cb c 3b ac a 3c ba b 1

eroberogeday lwm pln

- TMBr 144 -

KNitviTaCMuvijBiPBelak
!$0> eK[ a , b , c CabIcMnYnBitviCman .
cUrRsaybBaak;fa

a 2c
b 2a 2 c 2b
b2 .
c .
a 2 b 2 c2
3b
3c
3a
!$!> cMeBaHRKb;cMnYnBitviCman x , y , z cUrRsayfa
27
( x 2 yz )2
xy( xy z 2 )( zx y 2 )
8
!$@> eK[ a , b , c CacMnYnBitviCmanEdl abc 1 .
a2 .

a 3 b 3 c3 3
a
b
c

cUrRsayfa

4
bc ca ab
!$#> eK[ a , b , c CacMnYnBitviCmanEdl a b c 2 .
cUrRsayfa b(aa b) c(bb c) a(ac c) 2

!$$> RKb;cMnYnBitviCman a , b , c cUrRsayfa


a 2 ac
b 2 ba
c 2 cb
abc

2b a c 2c a b 2a b c
2
!$%> eK[ a , b , c IR . cUrRsayfa
a 2 ab b 2 b 2 bc c 2 c 2 ca a 2 9

2
2
2
4
(a b )
(b c )
(c a )

eroberogeday lwm pln

- TMBr 145 -

KNitviTaCMuvijBiPBelak
!$^> eK[cMnYnBitviCman a , b , c Edl a b c 1 .
cUrRsayfa
a
b
c
1

4b 3bc 4c 4c 3ca 4a 4a 3ab 4b 3


!$&> RKb;cMnYnBitviCman a , b , c Edl a b c 3 .
cUrRsayfa b ac 1 c ba 1 a bc 1 3
!$*> RKb;cMnYnBitviCman a , b , c Edl a b c 3 .

cUrRsayfa
a
b
c
3

2b 3c 1 2c 3a 1 2a 3b 1 4
!$(> eK[ a , b , c 0 . cUrRsaybBaak;fa
a3 b3
b 3 c3
c3 a3
6(ab bc ca)

(a b c) (a b)(b c)(c a)
a2 b 2
b2 c2
c2 a2

!%0> ebI a , b , c CacMnYnBitviCmanenaHcUrRsayfa


a 2 (b c ) b 2 (c a ) c 2 (a b )
2
2
abc
2
2
2
2
b c
c a
a b
!%!> ebI a , b , c CacMnYnBitviCmanenaHcUrRsayfa
a b c 9(a 2 b 2 c 2 )

b c a
(a b c )2

eroberogeday lwm pln

- TMBr 146 -

KNitviTaCMuvijBiPBelak
!%@> ebI a , b , c CacMnYnBitviCmanEdl ab bc ca abc 4
cUrRsayfa
1
1
1
3

(a 1)2 (b c) (b 1)2 (c a ) (c 1)2 (a b ) 8

!%#> ebI a , b , c CacMnYnBitviCmanEdl abc 1 . cUrRsayfa


1
1
1
3

(a 1)2 (b c) (b 1)2 (c a ) (c 1)2 (a b ) 8

!%$> eKman a , b , c , d CacMnYnBitminGviCman . cUrRsayfa


(a b c d ) 4 a(c d ) b(d a ) c(a b ) d(b c)
!%%> ebI a , b , c CacMnYnBitviCmanEdl abc 1 enaHcUrRsayfa
3

3 2
3 2
a2
b
c
3

bc
ca
ab
2
!%^> eK[ a , b , c CacMnYnBitviCman . cUrRsayfa
3

a2
b2
c2
2
2
1
2
2
2
2
a ab b b bc c c ca a
!%&> eK[ a , b , c CacMnYnBitviCman . cUrRsayfa
a b
c 1 1 1 1 ab
bc
ca

( 2 2 2

)
2
2
2
bc ca ab a b c 2 b c c a a b
!%*> ebI a , b , c CacMnYnBitminGviCman enaHcUrRsayfa
2(a 2 1)(b 2 1)(c 2 1) (a 1)(b 1)(c 1)(abc 1)

eroberogeday lwm pln

- TMBr 147 -

KNitviTaCMuvijBiPBelak
!%(> eK[ x , y , z 0 nig n IN . cUrRsayfa
( xn 3 xn 3)( y n 3 y n 3)( z n 3 z n 3) ( x y z )3

!^0> ebI a , b , c 1 cUrRsayfa


1 a2
1 b2
1 c2

2
2
2
2
1 b c 1 c a 1 a b

!^!> ebI a , b , c IR bgajfa a b c 1


!^@> ebI x , y , z CabIcMnYnBitviCmanenaHcUrRsayfa
4

a5 b 5 c5 1

xy
yz
zx
x
y
z

xy x2 y2 yz y2 z2 zx z2 x2 2x z 2y x 2z y

!^#> eK[ a , b , c CabIcMnYnBitminGviCman . cUrbgajfa


(a2 ab b2 )(b2 bc c2 )(c2 ca a2 ) (ab bc ca)3

!^$> eK[ a , b , c 0 Edl a b c 1 .


bgajfa ab cb bc ac ca ba 2 .
!^%> eK[GnuKmn_ f (x) x x Edl 0 , 0
1 .
k> RKb; x [ 0 , 1 ] bgajfa f (x) 0 .
x> RKb; u , v 0 Edl u v bgajfa u .v u v .
2

eroberogeday lwm pln

- TMBr 148 -

KNitviTaCMuvijBiPBelak
!^^> eK[ a , b , c , x , x , x , ..., x CacMnYnBitviCmanEdl
a b c 1 nig x .x .x .x .x 1 . cUrbgajfa
1

.
!^&> rgVg;carwkkgnRtIekaN ABC b:HRCug BC , CA , AB erogKaRtg;
AB
BC
CA
3
A , B , C .Rsayfa

AB
BC
CA
(ax1 bx1 c)(ax 2 bx 2 c)...(ax5 bx 5 c) 1
2

!^*> eK[ 0 x 2 . cUrRsayfa

1
1

1
1
2

sin n x
cos n x

, n IN *

!^(> eK[ a , b , c CaRCugnRtIekaNmYy . cUrRsaybBaak;fa


k> 3 bc ca ab) (a b c)2 4(bc ca ab)
abc
abc
(p 2
x> a2 b2 c2 36
) Edl p
35
p
2
K> 32 b a c c b a a c b 3
X> (p a)(p b)(p c) abc
8
!&0> RtIekaN ABC manRCug a , b , c ehIytag r CakaMrgVg;carwkkg
nig p CaknHbrimaRtnRtIekaN .
cUrRsayfa 1 2 1 2 1 2 12 .
(p a )

eroberogeday lwm pln

(p b )

(p c )

- TMBr 149 -

KNitviTaCMuvijBiPBelak
!&!> cUrRsaybBaak;fa
k> abc a2 (p a) b2 (p b) c2 (p c) 32 abc
x> bc(b c) ca(c a) ab(a b) 48(p a)(p b)(p c)
K> a3 (p a) b3 (p b) c3 (p c) abcp
Edl a , b , c CaRCugnRtIekaNmYy nig p CaknHbrimaRt .
!&@> ebI a , b , c CaRCugnRtIekaNmYyenaHcUrRsayfa
a 2b(a b ) b 2c(b c) c 2a(c a ) 0

etIeBlNaeTIbeK)ansmPaB .
!&#> eKtag a , b , c CaRCugnRtIekaNmYy ehIy p a b2 c .
cUrbgajfa
k> 64p3 (p a)(p b)(p c) 27a2b2c2
2p
1
1
1
2 2 2
x> abc
a

c
K> p 1 a p 1 b p 1 c p9
X> 154 pb ac pc ab ap bc 92
g> p p a p b p c 3p

eroberogeday lwm pln

- TMBr 150 -

KNitviTaCMuvijBiPBelak
!&$> eK[RtIekaN ABC mYymanRCug a , b , c . tag R CakaMrgVg;carwkkg
nRtIekaNenH .
k> Rsayfa a2 b2 c2 8R 2 (1 cos A cos B cos C)
x> edayeRbIRTwsIbTLibniccUrTajfa cos A cos B cos C 18 .
K> ebI A , B , C CamMuRsYcenaH
!&%> kgRtIekaN ABC mYycUrRsayfa
k> a cos A b cos B c cos C 2RS
x> sin A sin B sin C sin 2A sin 2B sin 2C

2 sin A sin B sin C

3 3
2

!&^> eK[RtIekaN ABC mYymanRCug a , b , c .


k> Rsayfa sin A2 2 abc rYcTajfa sin A2 sin B2 sin C2 18 .
x> tag r nig R CakaMrgVg;carwkkg nig carwkeRkAnRtIekaN ABC .
bgajfa sin A2 sin B2 sin C2 4rR rYcTajfa R 2r .
!&&> eK[RtIekaN ABC mYy .
cUrbgajfa cos A 2(cos B cos C) 2 ?
etIeBlNaeTIbsmPaBenHkayCasmPaB ?
eroberogeday lwm pln

- TMBr 151 -

KNitviTaCMuvijBiPBelak
!&*> kgRKb;RtIekaN ABC cUrRsayfa
2
cos A (cos B cos C) 1
2
!&(> eK[ x , y , z CacMnYnBitEdl xyz 0 .

RKb;cMnYnBit 0 .

cUrRsayfa x cos A y cos B z cos C 12 ( xyz yzx zxy )


Edl A , B , C CamMukgnRtIekaNmYy .
!*0> kgRKb;RtIekaN ABC cUrRsaybBaak;fa
k> cos2(A B) cos2(B C) cos2(C A) 24cosA cosB cosC
x> tan A2 tan B2 tan C2 3
K> tan2 A2 tan2 B2 tan2 C2 1
!*!> kgRKb;RtIekaN ABC cUrRsayfa
k> tan A2 tan B2 5 tan B2 tan C2 5

C
A
tan tan 5 4 3
2
2

x> tan6 A2 tan6 B2 tan6 C2 19


K> cot A cot B cot C 3
!*@> kgRKb;RtIekaN ABC cUrRsayfa
k> cot A2 cot B2 cot C2 3 3
eroberogeday lwm pln

- TMBr 152 -

KNitviTaCMuvijBiPBelak
x> cot 2 A2 cot 2 B2 cot 2 C2 9
!*#> kgRKb;RtIekaN ABC cUrRsayfa
1 cos A cos B cos C
3
sin A sin B sin C

etIeK)ansmPaBenAeBlNa ?
!*$> eK[RtIekaN ABC mYymanRCug a , b , c .
RsayfaebI a b 2 c enaH A B 2 C .
!*%> eK[RtIekaN ABC mYymanRCug a , b , c .
tag R CakaMrgVg;carwkkg nig S CapRklarbs;RtIekaNenH .
cUrRsaybBaak;fa a sin A b sin B c sin C 2 R3S
!*^> RtIekaN ABC mYymanmMu A B C .
tag R CakaMrgVg;carwkkg ehIy r CacmayrvagpitrgVg;carwkkg nig pit
rgVg;carwkeRkAnRtIekaN . cUrRsaybBaak;fa
2R cos A R d 2R cos B R d 2R cos C
!*&> eK[RtIekaN ABC mYymanRCug a , b , c .

tag R CakaMrgVg;carwkkgehIy r CakaMrgVg;carwkeRkAnRtIekaN .


cUrRsaybBaak;fa 9r a sin A b sin B c sin C 92R .
eroberogeday lwm pln

- TMBr 153 -

KNitviTaCMuvijBiPBelak
!**> eK[RtIekaN ABC mYymanRCug a , b , c .
tag S CapRklanRtIekaNenH . cUrRsaybBaak;fa
k> a2 b2 c2 4 3 S
x> a4 b4 c4 16S2
K> b2c2 c2a2 a2b2 16S2
X> 4 3S a 9abc
bc
g> (abc)2 ( 4S3 )3
!*(> eK[RtIekaN ABC mYymanRCug a , b , c .
tag S CapRklanRtIekaNenH . cUrRsaybBaak;fa
27(b 2 c 2 a 2 )2 (c 2 a 2 b 2 )2 (a 2 b 2 c 2 )2 (4S )6

!(0> eK[RtIekaN ABC mYymanRCug a , b , c .


a2 b 2 c2
tag S CapRklanRtIekaNenH ehIyyk u
2
nig v bc ca ab .
2
(u v )( 3u 5 v )
2 (u v )
.
Rsayfa
S
12
12
!(!> tag S CapRkLa nig a , b , c CaRCugnRtIekaNmYy .

RKb;

3 3 a n b n cn
n 0 bgajfa S

4
3

eroberogeday lwm pln

.
- TMBr 154 -

KNitviTaCMuvijBiPBelak
!(@> eK[RtIekaN ABC mYymanRCug a , b , c . eKtag r nig R
erogKaCakaMrgVg;carwkkg nig kaMrgVg;carwkeRkAnRtIekaN .
yk O CapitrgVg;carwkeRkA I CapitrgVg;carwkkg nig H CaGrtUsg;
nRtIekaN ABC . Rsayfa OH 2 9R 2 (a2 b2 c2 )
nig I H 2 2r 2 4R 2 cos A cos B cos C .
!(#> eKtag r nig R erogKaCakaMrgVg;carwkkg nig kaMrgVg;carwkeRkAnRtIekaN
mYyEdlman p CaknHbrimaRt . cUrRsayfa
k> 9r(r 4R ) 3p2 (4R r )2
x> 6r(r 4R ) 2p2 R 2 2(r 2R )2
K> 2p2 (2R r ) R(4R r )2
X> r(16R 5r ) p2 4R 2 4Rr 3r 2
!($> RtIekaN ABC mYymanRCug a , b , c eKtag p CaknHbrimaRt r nig R
erogKaCakaMrgVg;carwkkg nig kaMrgVg;carwkeRkAnRtIekaN. cUrRsayfa
k> p2 27r 2
x> 2p2 27rR
K> 36r 2 a2 b2 c2 9R 2
a2 b 2 c2
X> 3(2rR r )
r 2 2R 2
4
g> 36r 2 bc ca ab 9R 2
2

eroberogeday lwm pln

- TMBr 155 -

KNitviTaCMuvijBiPBelak
c> 5rR r 2 bc ca4 ab (r R )2
q> a(p a) b(p b) c(p c) 9Rr
C> abc 8rR 2 (12 3 16)Rr 2
Q> R3 a1 b1 1c 2r3
j> a1 b1 1c 2(3R 3r )
1

1
1
1
1

R 2 bc ca ab 4r 2
z> 4r 2 a abc
bc
D> abc (R 3 )3

d>

!(%> eK[RtIekaNsmBa ABC mYymanRCg BC a , AC b , AB c


eKtag p CaknHbrimaRt r CakaMrgVg;cawikkg R CakaMrgVg;cawikeRkA nig S
CapRkLarbs;RtIekaN ehIy rA , rB , rC tagerogKaCakaMrgVg;
carwkeRkARtIekaNkgmMuerogKa A , B , C .
cUrRsaybBaak;fa
a2
b2
c2
k> r r r r r r 4
B C
C A
A B
x> p a 4R 3rA

eroberogeday lwm pln

- TMBr 156 -

KNitviTaCMuvijBiPBelak
a 2 (b c)2 4R rA
3

2S
pa

K>
X> p2 rA 2 rB2 rC2
g> raA rbB rcC 3 8 3

rBrC rCrA rArB 3


c> bc ca ab 4
q> 9r rA rB rC 92 R
C> rA 2 rB2 rC2 274 R 2
Q> r 2 rA 2 rB2 rC2 7R 2

j> S 3 (R r )2
!(^> eK[RtIekaNsmBa ABC mYymanRCg BC a , AC b , AB c
eKtag p CaknHbrimaRt r CakaMrgVg;cawikkg R CakaMrgVg;cawikeRkA nig S
CapRkLarbs;RtIekaN ehIy ma , mb , mc tagerogKaCaemdan
nig ha , hb , hc Cakm<s;KUsecjBIkMBUlerogKa A , B , C .
cUrRsaybBaak;fa
k> 9r ma mb mc 92 R
x> 9r ha hb hc 92R
eroberogeday lwm pln

- TMBr 157 -

KNitviTaCMuvijBiPBelak
3
K> S 27 (ma mb mc )2
X> ma2 mb 2 mc2 3 3S

g> ma mb mc 4R r
!(&> eK[RtIekaNsmBa ABC mYymanRCg BC a , AC b , AB c
tag A , B , C erogKaCaknHbnat;BuHkgnmMu A , B , C nig S Ca
pRkLanRtIekaN . cUrRsayfa
k> A 2 B 2 C2 3 3S
x> 16( A4 B4 C4 ) 9(a4 b4 c4 )
K> A . B . C p.S Edl p a b2 c .
!(*> eKyk P CacMnucenAkgRtIekaN ABC ehIy D , E , F CaRbsBVrvag
AP , BP , CP CamYyRCug BC , CA , AB erogKa .
BE CF 9

.
cUrRsayfa AD
AP BP CP 2
!((> rgVg;carwkkgnRtIekaN ABC b:HnwgRCug BC , CA , AB erogKaRtg;
cMNuc D , E , F .
2

CA AB
Rsayfa BC


12
EF FD DE

eroberogeday lwm pln

- TMBr 158 -

KNitviTaCMuvijBiPBelak
@00> eK[RtIekaN ABC mYymankm<s; AD , BE , CF .
2

EF FD DE
3
Rsayfa BC



4
CA AB

@0!> eKmanRtIekaN ABC mYymanRCug a , b , c nigpRkla S

tag T CapRkLanRtIekaNsmgScarwkkgRtIekaN ABC xagelI .


2 3S 2

cUrRsayfa T 2 2 2
.
a b c 4 3S
@0@> eKmanRtIekaNBIr ABC nig XYZ carwkkgrgVg;EtmYy .
eKdwgfa AX // BC , BY // CA , CZ // AB .
tag a , b , c CaRCugn ABC nig x, y, z CaRCugn XYZ .
Rsayfa xa by cz 1 nig ax by zc 3 .
@0#> eK[RtIekaN ABC mYyEkgRtg; A nigmankm<s; AH h .
tag r CakaMrgVg;carwkkgnRtIekaN .
cUrRsayfa 2 1 hr 12 ?
@0$> eK[RtIekaN ABC mYyEkgRtg; A .
eKtag r CakaMrgVg;carwkkgnRtIekaN . cUrRsaybBaak;vismPaB
2
BC
r
AB.AC
.
etI
e
BlNaeTI
b
eK)ansmPaB
?
2
2
eroberogeday lwm pln

- TMBr 159 -

KNitviTaCMuvijBiPBelak
@0%> eKtag P CacMnucmYyenAkgRtIekaN ABC ehIyyk D , E , F CacMnuc
RbsBVrvagbnat; AP , BP , CP CamYyRCug BC, CA, AB erogKa .
BP CP
AP BP CP
cUrRsayfa AP

6 nig
.
.
8
PD PE PF
PD PE PF
@0^> eKtag A1 , B1 , C1 CabIcMnucsitenAelIRCug BC , CA , AB n ABC
1 BB1 CC1 3 .
cUrbgajfa 12 AA
AB BC CA
2
@0&> kgRtIekaN ABC mYycUrRsayfa
r
cos 2 A cos 2 B cos 2 B cos 2 C cos 2 C cos 2 A
1

R
cos A cos B
cos B cos C
cos C cos A

@0*> eKmanRtIekaN ABC mYyEkgRtg; A . tag M , N , P CacMnucb:H


rvagrgVg;carwkkgCamYyRCug BC, CA, AB . cUrRsayfa
4 3
MN MP
BC .
9
@0(> RtIekaN ABC mYycarwkkgrgVg;pit O kaM R .
tag R1 , R 2 , R 3 CakaMrgVg;carwkeRkAnRtIekaN OAB, OBC, OCA
erogKa . Rsayfa R1 R1 R1 3 R2 3 ?
1

@!0> ebI ABCD CaRbelLRkamenaHbgajfa | AB2 BC2 | AC.BD

eroberogeday lwm pln

- TMBr 160 -

KNitviTaCMuvijBiPBelak
@!!> ebI a , b , c CaRbEvgRCugnRtIekaNmYy ehIy ma , mb , mc Caemdan
nig R CakaMrgVg;carwkkgenaHbgajfa
k>

a2 b 2 b 2 c2 c2 a2

12R
mc
ma
mb

x> ma (bc a2 ) mb (ca b2 ) mc (ab c2 ) 0


@!@> (APMO, 1996) tag a, b, c CaRbEvgRCugnRtIekaNmYy . cUrbgajfa
abc bca cab a b c
@!#> (IMO, 1964) tag a, b, c CaRbEvgRCugnRtIekaNmYy . cUrbgajfa
a 2 (b c a ) b 2 (c a b ) c 2 (a b c) 3abc

@!$> (IMO, 1983) tag a, b, c CaRbEvgRCugnRtIekaNmYy . cUrbgajfa


a 2b(a b ) b 2c(b c) c 2a(c a ) 0 .
@!%> eK[RtIekaN ABC mYyEkgRtg; A . cMeBaHRKb;cMnYnKt; n 2
cUrRsayfa ABn ACn BCn .
@!^> tag ha , hb , hc CaRbEvgkm<s;nRtIekaN ABC mYyEdlcarwkeRkA
rgVg;pit I nigkaM r . cUrRsayfa
k> 1r h1 h1 h1
a

x> ha hb hc 9r

eroberogeday lwm pln

- TMBr 161 -

KNitviTaCMuvijBiPBelak
@!&> (Korea, 1995)eK[RtIekaN ABC mYyehIytag L , M , N
CacMnucsitelIRCug BC , CA , AB erogKa . tag P , Q nig R
CacMNucRbsBVrvagbnat; AL , BM nig CN CamYyrgVg;carwkeRkAn
AL BM CN

9 .
ABC erogKa . cUrRsayfa
LP MQ NR
@!*> (Shortlist IMO, 1997)RbEvgRCugnqekaN ABCDEF epgpat;
AB BC , CD DE nig EF FA .
DE FA 3
cUrRsayfa BC

.
BE DA FC 2
@!(> eKyk AD , BE , CF Cakm<s;nRtIekaN ABC ehIy PQ , PR, PS
CacmayBIcMnuc P eTARCug BC, CA, AB erogKa .
AD BE CF
cUrbgajfa PQ

9 .
PR PS
@@0> sntfargVg;carwkkgnRtIekaN ABC b:HeTAnwgRCug BC, CA, AD
p2
erogKaRtg; D, E, F . bgajfa EF FD DE 3
Edl p CaknHbrimaRtn ABC .
2

@@!> RtIekaN ABC mYymanRCug a , b , c nigtag S CapRklan ABC


cUrRsaybBaak;fa
k> a2 b2 c2 (a b)2 (b c)2 (c a)2 4 3S
x> ab bc ca 4 3S
eroberogeday lwm pln

- TMBr 162 -

KNitviTaCMuvijBiPBelak
a b c)abc
K> 3(ab
4 3S
bc ca
@@@> tag a , b , c CaRCug nig R CakaMrgVg;carwkeRkAnRtIekaNmYy .
a2
b2
c2
cUrRsayfa b c a c a b a b c 3 3R .
@@#> ctuekaNe)a:g ABCD mYymanRCug a , b , c , d . tag S CapRkLa

rbs;ctuekaNenH . cUrRsayfa
k> S ab 2 cd

ac bd
2
K> S a 2 c b 2 d

@@$> ebI P CacMnucenAkgRtIekaN ABC

x>

Edl PA , m PB, n PC

enaHcUrbgajfa (m mn n)( m n) a2 b2m c2n


Edl a , b , c CaRCugn ABC .
@@%> eKtag O CapitrgVg;carwkkg nig G CaTIRbCMuTmn;n ABC mYy .
eKtag r nig R erogKaCakaMrgVg;carwkkg nig carwkeRkAnRtIekaN .
(Balkan, 1996)
cUrRsayfa OG R(R 2r ) .

eroberogeday lwm pln

- TMBr 163 -

KNitviTaCMuvijBiPBelak
@@^> (Taiwan, 1997) eK[RtIekaN ABC mYycarwkkgrgVg;pit O nigkaM R
RsayfaebI AO kat;rgVg;carwkeRkAn OBC Rtg; D ehIy BO
kat;rgVg;carwkeRkAn OCA Rtg; E ehIy CO kat;rgVg;carwkeRkAn
OAB Rtg; F enaH OD.OE.PF 8R 3 .
@@&> (APMO, 1997)eK[ ABC CaRtIekaNmYy . bnat;BuHkgnmMu A
CYbGgt; BC Rtg; X nigCYbrgVg;carwkeRkAn ABC Rtg; Y .
AX
tag La AY
.eKkMNt; Lb nig Lc tamrebobdUcKaEdr .
cUrRsayfa

La

Lb

Lc

?
sin A sin B sin C
@@*> (Balkan, 1999)eK[ ABC CaRtIekaNmYymanmMukgCamMuRsYc ehIytag
L , M , N CaeCIgncMeNalEkgBITIRbCMuTmn; G nRtIekaN ABC
eTAelIRCug BC, CA, AB erogKa .
cUrbgajfa 274 SSLMN 14 ?
2

ABC

@@(> tag D nig E CacMnucenAelIRCug AB nig CA nRtIekaN ABC


edaydwgfa DE Rsbnwg BC ehIy DE b:HnwgrgVg;carwkkgn
AB BC CA
.
ABC .cUrbgajfa DE
8
(Italy, 1999)

eroberogeday lwm pln

- TMBr 164 -

KNitviTaCMuvijBiPBelak
@#0> (Mediterranean, 2000) tag P, Q, R, S CacMnuckNal erogKa
BC, CD, DA, AB nctuekaNe)a:g ABCD mYy .
cUrRsayfa
4( AP2 BQ2 CR2 DS2 ) 5( AB2 BC2 CD2 DA2 )

@#!> eKtag R nig r CakaMrgVg;carwkeRkA nig kaMrgVg;carwkkgn ABC .


sntfa A CamMuFMCageKnRtIekaN ABC .
yk M CacMNuckNaln BC ehIy X CaRbsBVrvagbnat;b:H
eTAnwgrgVg;carwkeRkARtIekaN ABC Rtg; B nig C .
cUrRsayfa Rr AM
(Korea, 2004)
AX
@#^> eK[RtIekaN ABC mYyehIyyk O CacMNucmYyenAkgRtIekaNenH .
bnat; OA, OB, OC CYbRCugnRtIekaNRtg; A1 , B1 , C1 erogKa .
tag R1 , R 2 , R 3 CakaMrgVg;carwkeRkAnRtIekaN OBC, OCA, OAB
erogKaehIy R CakaMnrgVg;carwkeRkAn ABC .
1 .R OB1 .R OC1 .R R
cUrRsayfa OA
1
2
3
AA
BB
CC
1

#@&>mMukgnRtIekaN ABC mYyCamMuRsYc ehIy ACB 2 ABC .


D CacMnucmYysitelIRCug BC ehIyepgpat; 2BAD ABC
1
1
1

cUrRsayfa BD
?
AB AC
( Mathematical Olympiad in Poland 1999 )

eroberogeday lwm pln

- TMBr 165 -

KNitviTaCMuvijBiPBelak
@#*> cMNuc D nig E sitenAelIRCug BC nig AC nRtIekaN ABC erogKa
bnat; AD nig BC kat;KaRtg; P .
K nig L CacMNucenAelIRCug BC nig AC erogKaEdl CLPK
BD
CaRbelLRkam . cUrRsayfa AE

EL DK
(Mathematical Olympiad in Poland 2000 )

@#(> bIcMNucxusKa A , B , C sitenAelIrgVg; (O) .


bnat;b:HrgVg; (O) Rtg; A nig B kat;KaRtg; P .
bnat;b:HrgVg; (O) Rtg; C kat;bnat; AB Rtg; Q .
cUrRsaybBaak;fa PQ 2 PB 2 QC2 ?
Mathematical Olympiad in Poland 2002 )
@$0>bnat;b:HeTAnwgrgVg;carwkkgnRtIekaNsmgS ABC kat;RCug AB
nig AC Rtg; D nig E .
AE
cUrRsaybBaak;fa AD

1 ?
DB EC
( Mathematical Olympiad in Poland 1995 )

@$!> bnat;BuHkgnmMu A , B , C nRtIekaN ABC mYykat;RCugQmRtg;


D, E, F erogKa ehIykat;rgVg;carwkeRkAnRtIekaN ABC Rtg;
AD BE CF
K , L, M erogKa . bgajfa

9 ?
DK EL FM
( Mathematical Olympiad in Poland 1996 )

eroberogeday lwm pln

- TMBr 166 -

KNitviTaCMuvijBiPBelak
@$@> kgRtIekaN ABC mYyman AB AC . D CacMNuckNaln BC
ehIy E sitenAelI AC . P nig Q CaeCIgncMeNalEkgBIcMNuc
B nig E erogKaeTelIbnat; AD .
bgajfa BE AE AC luHRtaEt AD PQ ?
(Mathematical Olympiad in Poland 1997 )

@$#>eKtag O CapitrgVg;carwkeRkA nig H CaGrtUsg;nRtIekaN ABC mYy


EdlmanmMukgCamMuRsYc .
cUrRsayfapRkLamYynRtIekaN AOH, BOH, COH esInwgplbUk
npRkLaBIrepSgeTot ?
(AsianPacific Mathematical Olympiad 2004 )

@$$> kgRtIekaN ABC cMNuc M nig N sitelIRCug AB nig AC erogKa


edaydwgfa MB BC CN .
tag r nig R erogKaCakaMrgVg;carwkkgnig carwkeRkAnRtIekaN .
cUrsresrkenSampleFob MN
CaGnuKmn_n R nig r .
BC
(AsianPacific Mathematical Olympiad 2005)

eroberogeday lwm pln

- TMBr 167 -

KNitviTaCMuvijBiPBelak
@$%> eK[ ABC CaRtIekaNmYymanmMukgCamMuRsYc ehIy BAC 60o
nig AB AC . yk I CapitrgVg;carwkkg nig H CaGrtUsg;
n ABC . cUrRsayfa 2AHI 3ABC ?
(AsianPacific Mathematical Olympiad 2007)

@$^> eKtag G CaTIRbCMuTmn;nRtIekaN ABC ehIy M CacMNuckNaln


RCug BC . bnat;KUsecjBI G Rsbnwg BC kat; AB Rtg; X ehIy
Ktat; AC Rtg; Y . sntfa XC nig GB kat;KaRtg; Q ehIy YB
nig GC kat;KaRtg; P .
cUrRsayfaRtIekaN MPQ dUcKanwgRtIekaN ABC ?
(AsianPacific Mathematical Olympiad 1991 )

@$&> elIRCug AB nig BC nkaer ABCD eKedAcMNuc E nig F erogKa


Edl BE BF . tag BN Cakm<s;nRtIekaN BCE .
cUrRsayfa DNF 90o ?
(AustrianPolish Mathematical Competition 1979)

@$*> rgVg;carwkkgnRtIekaN ABC b:HRCug BC nig AC Rtg; D nig E


erogKa . bgajfaebI AD BE enaH ABC CaRtIekaNsm)at ?
(Austrian Mathematical Olympiad 2006)

eroberogeday lwm pln

- TMBr 168 -

KNitviTaCMuvijBiPBelak
@$(> kgRtIekaN ABC mYyeKyk E CacMNuckNalnRCug AC nig F
CacMNuckNalnRCug BC . tag G CacMeNalEkgBI C eTA AB .
bgajfa EFG CaRtIekaNsm)atluHRtaEt ABC sm)at .
(Austrian Mathematical Olympiad 2008)

@%0> eKyk D, E nig F erogKaCacMNuckNalnRCug BC, CA nig AB


nRtIekaN ABC . yk Ha , Hb , Hc CaeCIgncMeNalEkgBI
A, B, C eTARCugQmerogKa .yk P, Q, R CacMNuckNaln
Hb H c H c H a nig Ha H b erogKa .
bgajfa PD, QE nig RF RbsBVKaRtg;cMnucmYy ?
(Austrian Mathematical Olympiad 2009)

@%!>kgctuekaNe)a:g ABCD mYyeKyk E CacMnucRbsBVrvagGgt;RTUg


ehIy S1 , S2 nig S CapRkLan ABE , CDE nig ABCDerogKa
cUrRsayfa S1 S2 S ?
(Austrian Mathematical Olympiad 1990)

@%@> bBaekaNe)a:g ABCDE mYycarwkkgrgVg; . cmayBI A eTAbnat;


BC, CD, DE esIerogKa a, b, c .
cUrKNnacmayBIkMBUl A eTAbnat; (BE) .
(Austrian Mathematical Olympiad 1990)

eroberogeday lwm pln

- TMBr 169 -

KNitviTaCMuvijBiPBelak
@%#> eK[RtIekaNsm)at ABC mYyEdl A 90o .
eKyk M CacMNuckNaln AB . bnat;mYyKUsecjBI A ehIyEkg
nwg CM kat;RCug BC Rtg; P . bgajfa AMC BMP ?
(Baltic Way 2000)

@%$>eK[RtIekaN ABC mYymanmMu A 120o . cMNuc K nig L


sitenAelIRCug AB nig AC erogKa .
eKsg;RtIekaNsmgS BKP nig CLQ enAeRkARCugnRtIekaN ABC
cUrRsayfa PQ 23 ( AB AC) ?
(Baltic Way 2000)

@%%> kgRtIekaN ABC mYyeK[ 2AB AC BC . cUrRsayfapitrgVg;


carwkkg ABC pitrgVg;carwkeRkA ABC ehIycMNuckNaln
AC nig BC sitenAelIrgVg;EtmYy .
(Problems for the Team Competition Baltic Way 1999)

@%^>eK[ ABC nig DAC CaRtIekaNsm)atBIrEdlmanmuMkMBUl


BAC 20o nig ADC 100o .
cUrRsayfa AB BC CD ?
(Flanders Mathematical Olympiad 1996)

eroberogeday lwm pln

- TMBr 170 -

KNitviTaCMuvijBiPBelak
@%&>eKtag R CakaMrgVg;carwkeRkA nig r CakaMrgVg;carwkkgn ABC mYy.
S CacMnucmYyenAkg ABC ehIybnat; AS, BS, CS kat;RCugQm
Rtg; X, Y, Z erogKa .
CY.AY AZ.BZ R r
cUrRsayfa BX.CX

2
2
2
r
AX

BY

CZ

( Bosnia and Hercegovina Mathematical Olympiad 2000 )


o

@%*> eK[RtIekaNEkgmYy ABC Edl ACB 90 ehIyyk


CH Edl H AB CakMBs;cMeBaHRCug AB ehIytag P nig Q
CacMNucb:HrvagrgVg;carwkkgnRtIekaN ABC CamYyRCug AC nig BC
erogKa . ebI AQ HP cUrkMNt;pleFob AH
?
BH

(Bulgarian Mathematical Olympiad 2006)

@%(> eK[ctuekaNcarwkkgrgVg; ABCD mYy. bnat; AD nig BC CYbKa


Rtg;cMNuc E ehIyGgt;RTUg AC nig BD CYbKaRtg; F .
ebI M nig N CacMNuckNaln AB nig CD enaHcUrbgajfa
MN 1 AB CD

.
EF 2 CD AB
(Bulgarian Mathematical Olympiad 1997)

eroberogeday lwm pln

- TMBr 171 -

KNitviTaCMuvijBiPBelak
@^0> bBaekaNe)a:g ABCDE mYycarwkkgrgVg;kaM R . eKsMKal; rXYZ Ca
rgVas;kaMnrVg;carwkkgn XYZ . cUrRsaybBaak;fa
k> cos CAB cos ABC cos BCA 1 rABC
R
x> ebI rABC rAED nig rABD rAEC enaH ABC AED
(Bulgarian Mathematical Olympiad 1998)

@^!> eK[bnat; () kat;RCug AC nig BC nRtIekaN ABC erogKaRtg;


cMNuc E nig F .
cUrRsayfabnat; () kat;tampitrgVg;carwkkgnRtIekaN ABC
AE
BF
luHRtaEt BC. CE
AC.
AB .
CF
(Bulgarian Mathematical Olympiad 1973)

@^@> eKtag M CaTIRbCMuTmn;nRtIekaN ABC mYy .


cUrRsayfavismPaB sin CAM sin CBM

2
3

(Bulgarian Mathematical Olympiad 1997)

@^#> ebI a , b , c CaRCug nig R CakaMrgVg;carwkeRkAnRtIekaN ABC


enaHbgajfa R

a2 b2
2 2a 2b c
2

etIsmPaBmanenAeBlNa?
eroberogeday lwm pln

.
( Baltic Way 1998 )

- TMBr 172 -

KNitviTaCMuvijBiPBelak

ksaeyag
!>

101 problems in Algebra


( Titu Andreescu & Zuming Feng)

@>

Selected Problems of the Vietnamese Mathematical


Olympiad

(19622009)

( Le Hai Chau & Le Hai Khoi )

#>
$>

TOPICS IN INEQUALITIES

( Hojoo Lee )

Mathematical Olympiad in China


(Xiong Bin & Lee Peng Yee )

%>

Lecture Notes onMathematical Olympiad Courses


( Xu Jiagu )

^>

INFINITY

&>

(Hojoo Lee, Tom Lovering, and Cosmin Pohoat )


Secrets in Inequalities
(PHAM KIM HUNG)

eroberogeday lwm pln

- TMBr 173 -

You might also like